*NURSING > TEST BANK > Fundamentals of Nursing 9th edition - test bank (complete A+ guide) latest 2020. (All)

Fundamentals of Nursing 9th edition - test bank (complete A+ guide) latest 2020.

Document Content and Description Below

Fundamentals of Nursing 9th edition - test bank Chapter 02: The Health Care Delivery System 1. The nurse is caring for a patient whose insurance coverage is Medicare. The nurse should consider which... information when planning care for this patient? a. Capitation provides the hospital with a means of recovering variable charges. b. The hospital will be paid for the full cost of the patient’s hospitalization. c. Diagnosis-related groups (DRGs) provide a fixed reimbursement of cost. d. Medicare will pay the national average for the patient’s condition. 2. A nurse is teaching the staff about managed care. Which information should the nurse include in the teaching session? a. Managed care insures full coverage of health care costs. b. Managed care only assumes the financial risk involved. c. Managed care allows providers to focus on illness care. d. Managed care causes providers to focus on prevention. 3. A nurse is teaching a family about health care plans. Which information from the nurse indicates a correct understanding of the Affordable Care Act? a. A family can choose whether to have health insurance with no consequences. b. Primary care physician payments from Medicaid services can equal Medicare. c. Adult children up to age 26 are allowed coverage on the parent’s plan. d. Private insurance companies can deny coverage for any reason. 4. A nurse is caring for a patient in the hospital. When should the nurse begin discharge planning? a. When the patient is ready b. Close to the time of discharge c. Upon admission to the hospital d. After an order is written/prescribed 5. The nurse is applying for a position with a home care organization that specializes in spinal cord injury. In which type of health care facility does the nurse want to work? a. Secondary acute b. Continuing c. Restorative d. Tertiary 6. A nurse provides immunization to children and adults through the public health department. Which type of health care is the nurse providing? a. Primary care b. Preventive care c. Restorative care d. Continuing care 7. A nurse is following the PDSA cycle for quality improvement. Which action will the nurse take for the letter “A”? a. Act b. Alter c. Assess d. Approach 8. The nurse is trying to determine how well a certain health plan compares with other health plans. To gather this type of data, which information will the nurse utilize? a. Pew Health Professions Commission b. Healthcare Effectiveness Data and Information Set (HEDIS) c. American Nurses Credentialing Center (ANCC) Magnet Recognition Program d. Hospital Consumer of Assessment of Healthcare Providers and Systems (HCAHPS) 9. An older adult patient has extensive wound care needs after discharge from the hospital. Which facility should the nurse discuss with the patient? a. Hospice b. Respite care c. Assisted living d. Skilled nursing 10. A nurse working in a community hospital’s emergency department provides care to a patient having chest pain. Which level of care is the nurse providing? a. Continuing care b. Restorative care c. Preventive care d. Tertiary care 11. A nurse is teaching about the effects of globalization. Which information should the nurse include in the teaching session? a. Increased spread of communicable diseases b. Increased homogeneous mix of nursing staff c. Decreased poverty and increased “health tourism” d. Decreased urbanization as populations shift to the suburbs 12. A nurse is using research findings to improve clinical practice. Which technique is the nurse using? a. Performance improvement b. Integrated delivery networks c. Nursing-sensitive outcomes d. Utilization review committees 13. Which finding indicates the best quality improvement process? a. Staff identifies the wait time in the emergency department is too long. b. Administration identifies the design of the facility’s lobby increases patient stress. c. Director of the hospital identifies the payment schedule does not pay enough for overtime. d. Health care providers identify the inconsistencies of some of the facility’s policy and procedures. 14. A nurse is providing home care to a home-bound patient treated with intravenous (IV) therapy and enteral nutrition. What is the home health nurse’s primary objective? a. Screening b. Education c. Dependence d. Counseling 15. A nurse hears a co-worker state that anybody could be a nurse since it is so automated with infusion devices and electronic monitoring; technology is doing the work. What is the nurse’sbest response? a. “Technology use has to be combined with nursing judgment.” b. “The focus of effective nursing care is technology.” c. “If it’s so easy, why don’t you do it?” d. “That is true in the 20th century.” 16. A nurse is completing a minimum data set. Which area is the nurse working? a. Nursing center b. Psychiatric facility c. Rehabilitation center d. Adult day care center MULTIPLE RESPONSE 1. Which government-instituted programs should the nurse include in a teaching session about controlling health care costs? (Select all that apply.) a. Professional standards review organizations b. Prospective payment systems c. Diagnosis-related groups d. Third-party payers e. “Never events” , B, C 2. A nurse is teaching the staff about the Institute of Medicine competencies. Which examples indicate the staff has a correct understanding of the teaching? (Select all that apply.) a. Use informatics. b. Use transparency. c. Apply globalization. d. Apply quality improvement. e. Use evidence-based practice. , D, E 3. A nurse is evaluating care based upon the nursing quality indicators. Which areas should the nurse evaluate? (Select all that apply.) a. Patient satisfaction level b. Hospital readmission rates c. Nursing hours per patient day d. Patient falls/falls with injuries e. Value stream analysis for quality , C, D 4. A nurse is working in a health care organization that has achieved Magnet status. Which components are indicators of this status? (Select all that apply.) a. Empirical quality results b. Structural empowerment c. Transformational leadership d. Exemplary professional practice e. Willingness to recommend the agency , B, C, D MATCHING A nurse is teaching about the different types of health care plans. Match the correct information to the type of health care plans the nurse should include in the teaching session. a. Insurance for low-income families b. Federal insurance for people aged 65 and older c. Health maintenance focus to specific group of voluntarily enrolled people d. Services at a discount for companies under contract 1. Preferred provider organization 2. Managed care organization 3. Medicaid 4. Medicare 1.ANSDIF:Understand (comprehension)REF:15-16 OBJ: Compare the various methods for financing health care. TOP: Teaching/Learning MSC:Management of Care 2.ANS:CDIF:Understand (comprehension)REF:15-16 OBJ: Compare the various methods for financing health care. TOP: Teaching/Learning MSC:Management of Care 3.ANS:ADIF:Understand (comprehension)REF:15-16 OBJ: Compare the various methods for financing health care. TOP: Teaching/Learning MSC:Management of Care 4.ANS:BDIF:Understand (comprehension)REF:15-16 OBJ: Compare the various methods for financing health care. TOP: Teaching/Learning MSC:Management of Care Free Chapter 03: Community-Based Nursing Practice 1. A nurse is working as a public health nurse. What will be the nurse’s primary focus? a. The individual as one member of a group b. Individuals and families c. Needs of a population d. Health promotion 2. A nurse wants to become a specialist in public health nursing. Which educational requirement will the nurse have to obtain? a. A baccalaureate degree in nursing b. Preparation at the basic entry level c. The same level of education as the community health nurse d. A graduate level education with a focus in public health science 3. A nurse is working as a community health nurse. Which action is a priority for this nurse? a. Provide direct care to subpopulations. b. Focus on the needs of the ill individual. c. Provide first level of contact to health care systems. d. Focus on providing care in various community settings. 4. A nurse is focusing on acute and chronic care of individuals and families within a community while enhancing patient autonomy. Which type of nursing care is the nurse providing? a. Public health b. Community health c. Community-based d. Community assessment 5. The community health nurse is administering flu shots to children at a local playground. What is the rationale for this nurse’s action? a. To prevent individual illness b. To prevent community outbreak of illness c. To prevent outbreak of illness in the family d. To prevent needs of the local population groups 6. A nurse attended a seminar on community-based health care. Which information indicates the nurse has a good understanding of community-based health care? a. It occurs in hospitals. b. Its focus is on ill individuals. c. Its priority is health promotion. d. It provides services primarily to the poor. 7. A nurse is using the Healthy People 2020 to establish goals for the community. Which goal ispriority? a. Reduce health care costs. b. Increase life expectancy. c. Provide services close to where patients live. d. Isolate patients to prevent the spread of disease. 8. A nurse is working in community-based nursing. Which competency is priority for this nurse? a. Caregiver b. Collaborator c. Change agent d. Case manager 9. A nurse observes an outbreak of lice in a certain school district. The nurse collects data and identifies a common practice of sharing lockers, caps, and hair brushes. The nurse shares the information with the school. Which community-based nursing competency did the nurse use? a. Educator b. Caregiver c. Case manager d. Epidemiologist 10. A nurse is providing screening at a health fair. Which finding indicates the person may be a vulnerable patient who is most likely to develop health problems? a. One who is pregnant b. One who has excessive risks c. One who has unlimited access to health care d. One who uses nontraditional healing practices 11. The instructor is teaching student nurses about identifying members of vulnerable populations when the nursing student asks, “Why is it that not all poor people are considered members of vulnerable populations?” How should the nurse respond? a. “All poor people are members of a vulnerable population.” b. “Poor people are members of a vulnerable population only if they take drugs.” c. “Poor people are members of a vulnerable population only if they are homeless.” d. “Members of vulnerable groups frequently have a combination of risk factors.” 12. The nurse is making a home visit to a Korean mother after the birth of girl. The spouse is pressing different parts of the patient’s hand and lower arm to relieve a headache. What is the nurse’s next action? a. Tell the spouse to stop and give the mother acetaminophen. b. Let the spouse finish and then give the mother medication. c. Ask the mother and/or spouse to explain the procedure. d. Explain to the spouse that it will not work. 13. A nurse is assessing the social system of a community. Which area should the nurse assess? a. Housing b. Economic status c. Volunteer programs d. Predominant ethnic groups 14. The nurse is working with a 16-year-old pregnant female who tells the nurse that she needs an abortion. The nurse, acting as a counselor, provides the patient with information on alternatives to abortion, but after several sessions, the patient still insists on having the abortion. What should the nurse, in the counselor role, do next? a. Encourage the patient to speak with a “Right-to-Life” advocate. b. Refuse to provide a referral to an abortion service. c. Provide referral to an abortion service. d. Delay referral to an abortion service. 15. Before a patient with beginning stage of Alzheimer’s disease is discharged, the community-based nurse is making a visit to the patient’s home. The patient’s daughter and family live in the home with the patient. What is the major focus of this visit? a. Teach the family how to monitor blood pressure. b. Demonstrate techniques for providing care. c. Stress to the family how difficult it will be to provide care at home. d. Encourage the family to send the patient to an extended care facility. 17. The nurse uses statistics on increased incidence of communicable disease to influence legislatures to pass a bill for mandatory vaccinations to enroll in school. Which type of nursing will the nurse use in this process? a. Public health nursing b. Community-based nursing c. Community health nursing d. Vulnerable population nursing MULTIPLE RESPONSE 1. A community-based nursing is working with a family. For which key areas will the nurse need a strong knowledge base? (Select all that apply.) a. Family theory b. Communication c. Group dynamics d. Cultural diversity e. Individual-centered care , B, C, D 2. Which community-based nursing activities indicate the nurse is working in the role of educator? (Select all that apply.) a. Offers prenatal classes b. Offers a child safety program c. Offers to defend patients’ decisions d. Offers creative solutions to local problems e. Offers coordinate resources after discharge , B 3. A nurse is caring for vulnerable populations in a local community. Which patients will the nurse care for in this community? (Select all that apply.) a. A 47-year-old immigrant who speaks only Spanish b. A 35-year-old living in own home c. A 22-year-old pregnant woman d. A 40-year-old schizophrenic e. A 15-year-old rape victim , D, E MATCHING A nurse is assessing a community. Match each community element the nurse will assess with the correct example. a. Education level b. Housing c. Government 1. Structure 2. Population 3. Social system 1.ANS:BDIF:Understand (comprehension)REF:38 OBJ: Describe elements of a community assessment. TOP: Assessment MSC:Health Promotion and Maintenance 2.ANS:ADIF:Understand (comprehension)REF:38 OBJ: Describe elements of a community assessment. TOP: Assessment MSC:Health Promotion and Maintenance 3.ANS:CDIF:Understand (comprehension)REF:38 OBJ: Describe elements of a community assessment. TOP: Assessment MSC:Health Promotion and Maintenance Chapter 04: Theoretical Foundations of Nursing Practice 1. The nursing instructor is teaching a class on nursing theory. One of the students asks, “Why do we need to know this stuff? It doesn’t really affect patients.” What is the instructor’s bestresponse? a. “You are correct, but we have to learn it anyway.” b. “This keeps the focus of nursing narrow.” c. “Theories help explain why nurses do what they do.” d. “Exposure to theories will help you later in graduate school.” 2. The nurse is caring for a patient who does not follow the prescribed regimen for diabetes management. As a prescriber to Orem’s theory, the nurse interviews the patient in an attempt to identify the cause of the patient’s “noncompliance.” What is the rationale for the nurse’s behavior? a. Orem’s theory is useful in designing interventions to promote self-care. b. Orem’s theory focuses on cultural issues that may affect compliance. c. Orem’s theory allows for reduction of anxiety with communication. d. Orem’s theory helps nurses manipulate the patient’s environment. 3. A nurse is testing meditation for migraine headaches and the expected outcome of care when performing this intervention. Which type of theory is the nurse using? a. Grand b. Prescriptive c. Descriptive d. Middle-range 4. The nurse researcher is evaluating whether holding pressure at an injection site after injecting the anticoagulant enoxaparin will reduce bruising at the injection site. This study involves a prescriptive theory. What is the nurse’s rationale for involving a prescriptive theory? a. It explains why bruising occurs. b. It is broad in scope and complex. c. It tests a specific nursing intervention. d. It reflects a wide variety of nursing care situations. 5. A nurse is using nursing theory and the nursing process simultaneously to plan nursing care. How will the nurse use nursing theory and the nursing process in practice? a. Nursing theory can direct how a nurse uses the nursing process. b. Nursing theory requires the nursing process to develop knowledge. c. Nursing theory with the nursing process has a minor role in professional nursing. d. Nursing theory combined with the nursing process is specific to certain ill patients. 6. The nurse views the patient as an open system that needs help in coping with stressors. Which theorist is the nurse using? a. King b. Levine c. Neuman d. Johnson 7. The nurse is caring for a patient diagnosed with essential hypertension. The health care provider prescribes blood pressure medication that the nurse administers. The nurse then monitors the patient’s blood pressure for several days to help determine effectiveness. Which system component is the nurse evaluating? a. Input b. Output c. Content d. Feedback 8. A patient is admitted with possible methicillin-resistant Staphylococcus aureus (MRSA) and is placed in isolation until cultures can be obtained and declared noninfectious. During the isolation process, the nurse encourages family visits. Which level of Maslow’s hierarchy of needs is the nurse promoting when the family is encouraged to visit? a. First level b. Second level c. Third level d. Fourth level 9. A nurse is caring for pediatric patients and using the developmental theory to plan nursing care. What is the focus of this nurse’s care? a. Humans have an orderly, predictive process of growth and development. b. Humans respond to threats by adapting with growth and development. c. Humans respond with cognitive principles for growth and development. d. Humans have psychosocial domains to growth and development. 10. Upon assessment, the nurse notices that the patient’s respirations have increased, and the tip of the nose and earlobes are becoming cyanotic. The nurse finds that the patient’s pulse rate is over 100 beats per minute. According to Maslow’s hierarchy of needs, which patient need should the nurse address first? a. Self-esteem b. Physiological c. Self-actualization d. Love and belonging 11. Which behavior from a nurse indicates the nurse is using Nightingale’s theory to plan nursing care? a. Knows all about the disease processes affecting patients b. Focuses on medication administration and treatments c. Thinks about the patients and patients’ environments d. Considers nursing knowledge and medicine the same 12. The home health nurse listens to the patient’s concerns about having “open-heart” surgery. The nurse explains the different surgical procedures and other options, like cardiac rehabilitation. After several visits, the patient wants cardiac rehabilitation. The nurse notifies the health care provider and sets up a referral. Which theory is the nurse using? a. Peplau’s theory b. Henderson’s theory c. Nightingale’s theory d. Orem’s self-care deficit theory 13. The nurse is caring for a patient who is actively bleeding. The health care provider prescribes blood transfusions. The patient is a Jehovah’s Witness and does not want blood products. The nurse contacts the health care provider to request alternative treatment. Which theory is the nurse using? a. Roy’s theory b. Leininger’s theory c. Watson’s theory d. Orem’s theory 14. The patient is terminally ill and is receiving hospice care. The nurse cares for the patient by bathing, shaving, and repositioning him. The patient would like a Catholic priest called to provide the Sacrament of the Sick. The nurse places a call and arranges for the priest’s visit. Which theory does this nurse’s care represent? a. Roy’s theory b. Watson’s theory c. Henderson’s theory d. Orem’s self-care deficit theory 15. The patient is newly diagnosed with diabetes and will be discharged in the next day or so. The nurse is teaching the patient how to draw up and self-administer insulin. Which nursing theory is the nurse utilizing? a. Watson’s theory b. Orem’s theory c. Roger’s theory d. Henderson’s theory 16. A nurse is conducting research about the needs of depressed patients. The nurse writes the following: Depression is a patient reporting a score above 7 on the Hamilton Depression Rating Scale. What did the nurse write? a. Operational definition b. Conceptual definition c. Paradigm d. Concept 17. Which action indicates the nurse is using the nursing process in patient care? a. Generates nursing knowledge for use in nursing practice. b. Conceptualizes an aspect of nursing to predict nursing care. c. Develops nursing care as a specific, distinct phenomenon. d. Delivers nursing care using a systematic approach. 18. A nurse is using theoretical knowledge in nursing practice to provide patient care. Which nursing behavior is an example of theoretical knowledge? a. Reads about different concepts b. Reflects on clinical experiences c. Combines the art and science of nursing d. Creates a narrow understanding of nursing practice 19. A nurse is using Maslow’s hierarchy of needs to prioritize care. Place the levels in order of basic priority to highest priority that the nurse will follow. 1. Physiological 2. Self-esteem 3. Self-actualization 4. Safety and security 5. Love and belonging a. 4, 1, 2, 3, 5 b. 1, 4, 5, 3, 2 c. 4, 5, 3, 2, 1 d. 1, 4, 5, 2, 3 MULTIPLE RESPONSE 1. A nurse is using a nursing metaparadigm to define nursing. Which concepts will the nurse include? (Select all that apply.) a. Person b. Disease c. Health d. Nursing e. Environment , C, D, E 2. A nurse wants to incorporate psychosocial theories into nursing practice. Which elements will the nurse include? (Select all that apply.) a. Physiological needs of the patient b. Psychological needs of the patient c. Sociocultural needs of the patient d. Cognitive needs of the patient e. Spiritual needs of the patient , B, C, E Chapter 05: Evidence-Based Practice 1. A nurse uses evidence-based practice (EBP) to provide nursing care. What is the bestrationale for the nurse’s behavior? a. EBP is a guide for nurses in making clinical decisions. b. EBP is based on the latest textbook information. c. EBP is easily attained at the bedside. d. EBP is always right for all situations. 2. In caring for patients, what must the nurse remember about evidence-based practice (EBP)? a. EBP is the only valid source of knowledge that should be used. b. EBP is secondary to traditional or convenient care knowledge. c. EBP is dependent on patient values and expectations. d. EBP is not shown to provide better patient outcomes. 3. A nurse wants to change a patient procedure. Which action will the nurse take to easily find research evidence to support this change? a. Read all the articles found on the Internet. b. Make a general search of the Internet. c. Use a PICOT format for the search. d. Start with a broad question. 4. A nurse has collected several research findings for evidence-based practice. Which article will be the best for the nurse to use? a. An article that uses randomized controlled trials (RCT) b. An article that is an opinion of expert committees c. An article that uses qualitative research d. An article that is peer-reviewed 5. The nurse is reviewing a research article on a patient care topic. Which area should entice the nurse to read the article? a. Literature review b. Introduction c. Methods d. Results 6. The nurse is caring for a patient with chronic low back pain. The nurse wants to determine the best evidence-based practice regarding clinical guidelines for low back pain. What is thebest database for the nurse to access? a. MEDLINE b. EMBASE c. PsycINFO d. AHRQ 7. A nurse writes the following PICOT question: How do patients with breast cancer rate their quality of life? How should the nurse evaluate this question? a. A true PICOT question regardless of the number of elements b. A true PICOT question because the intervention comes before the control c. Not a true PICOT question because the comparison comes after the intervention d. Not a true PICOT question because the time is not designated 8. A nurse is reviewing literature for an evidence-based practice study. Which study should the nurse use for the most reliable level of evidence that uses statistics to show effectiveness? a. Meta-analysis b. Systematic review c. Single random controlled trial d. Control trial without randomization The main difference is that in a meta-analysis the researcher uses statistics to show the effect of an intervention on an outcome. In a systematic review no statistics are used to draw conclusions about the evidence. A single random controlled trial (RCT) is not as conclusive as a review of several RCTs on the same question. Control trials without randomization may involve bias in how the study is conducted. 9. A nurse is reviewing research studies for evidence-based practice. Which article should the nurse use for qualitative nursing research? a. An article about the number of falls after use of no side rails b. An article about infection rates after use of a new wound dressing c. An article about the percentage of new admissions on a new floor d. An article about emotional needs of dying patients and their families 10. A nurse develops the following PICOT question: Do patients who listen to music achieve better control of their anxiety and pain after surgery when compared with patients who receive standard nursing care following surgery? Which information will the nurse use as the “C”? a. After surgery b. Who listen to music c. Who receive standard nursing care d. Achieve better control of their anxiety and pain 11. The nurse uses a PICOT question to develop an evidence-based change in protocol for a certain nursing procedure. However, to make these changes throughout the entire institution would require more evidence than is available at this time. What is the nurse’s best option? a. Conduct a pilot study to investigate findings. b. Drop the idea of making the change at this time. c. Insist that management hire the needed staff to facilitate the change. d. Seek employment in another institution that may have the staff needed. 12. The nurse is trying to identify common general themes relative to the effectiveness of cardiac rehabilitation from patients who have had heart attacks and have gone through cardiac rehabilitation programs. The nurse conducts interviews and focus groups. Which type of research is the nurse conducting? a. Nonexperimental research b. Experimental research c. Qualitative research d. Evaluation research 13. In conducting a research study, the nurse researcher guarantees the subject no information will be reported in any manner that will identify the subject and only the research team will have access to the information. Which concept is the nurse researcher fulfilling? a. Bias b. Confidentiality c. Informed consent d. The research process 14. The nurse researcher is preparing to publish the findings and is preparing to add the limitations to the manuscript. Which area of the manuscript will the nurse researcher add this information? a. Abstract b. Conclusion c. Study design d. Clinical implications 15. A nurse is trying to decrease the rate of falls on the unit. After reviewing the literature, a strategy is implemented on the unit. After 3 months, the nurse finds that the falls have decreased. Which process did the nurse institute? a. Performance improvement b. Peer-reviewed project c. Generalizability study d. Qualitative research 16. A nurse identifies a clinical problem with pressure ulcers. Which step should the nurse take next in the research process? a. Analyze results. b. Conduct the study. c. Determine method. d. Develop a hypothesis. 17. After reviewing the literature, the evidence-based practice committee institutes a practice change that bedrails should be left in the down position and hourly nursing rounds should be conducted. The results indicate over a 40% reduction in falls. What is the committee’s next step? a. Evaluate the changes in 1 month. b. Implement the changes as a pilot study. c. Wait a month before implementing the changes. d. Communicate to staff the results of this project. 18. A nurse is developing a care delivery outcomes research project. Which population will the nurse study? a. Nurses b. Patients c. Administrators d. Health care providers 19. A nurse is implementing an evidence-based practice project regarding infection rates. After reviewing research literature, which other evidence should the nurse review? a. Quality improvement data b. Inductive reasoning data c. Informed consent data d. Biased data When implementing an evidence-based practice project, it is important to first review evidence from appropriate research and quality improvement data. Inductive reasoning is used to develop generalizations or theories from specific observations; this study needs specifics. Informed consent is not data but a process and form that subjects must sign before participating in research projects/studies. Biased data is based on opinions; facts are needed for this study. DIF:Understand (comprehension)REF:61 OBJ: Explain the relationship between evidence-based practice and performance improvement. TOP:ImplementationMSC:Management of Care 20. A nurse is using the research process. Place in order the sequence that the nurse will follow. 1. Analyze results. 2. Conduct the study. 3. Identify clinical problem. 4. Develop research question. 5. Determine how study will be conducted. a. 3, 4, 5, 2, 1 b. 4, 3, 5, 2, 1 c. 3, 5, 4, 2, 1 d. 4, 5, 3, 2, 1 MULTIPLE RESPONSE 1. The nurse is preparing to conduct research that will allow precise measurement of a phenomenon. Which methods will provide the nurse with the right kind of data? (Select all that apply.) a. Surveys b. Phenomenology c. Grounded theory d. Evaluation research e. Nonexperimental research , D, E 2. Before conducting any study with human subjects, the nurse researcher must obtain informed consent. What must the nurse researcher ensure to obtain informed consent? (Select all that apply.) a. Gives complete information about the purpose b. Allows free choice to participate or withdraw c. Understands how confidentiality is maintained d. Identifies risks and benefits of participation e. Ensures that subjects complete the study , B, C, D 3. The nurse is reviewing nursing research literature related to a potential practice problem on the nursing unit. What is the rationale for the nurse’s action? (Select all that apply.) a. Nursing research ensures the nurse’s promotion. b. Nursing research identifies new knowledge. c. Nursing research improves professional practice. d. Nursing research enhances effective use of resources. e. Nursing research leads to decreases in budget expenditures. , C, D Chapter 06: Health and Wellness 1. A nurse is teaching about the goals of Healthy People 2020. Which information should the nurse include in the teaching session? a. Eliminate health disparities in America. b. Eliminate health behaviors in America. c. Eliminate quality of life in America. d. Eliminate healthy life in America. 2. A nurse is following the goals of Healthy People 2020 to provide care. Which action should the nurse take? a. Allow people to continue current behaviors to reduce the stress of change. b. Focus only on health changes that will lead to better local communities. c. Create social and physical environments that promote good health. d. Focus on illness treatment to provide fast recuperation. 3. A nurse is using the World Health Organization definition of health to provide care. Which area will the nurse focus on while providing care? a. Making sure the patients are disease free b. Making sure to involve the whole person c. Making sure care is strictly personal in nature d. Making sure to focus only on the pathological state 4. The nurse is preparing a smoking cessation class for family members of patients with lung cancer. The nurse believes that the class will convert many smokers to nonsmokers once they realize the benefits of not smoking. Which health care model is the nurse following? a. Health belief model b. Holistic health model c. Health promotion model d. Maslow’s hierarchy of needs 5. A nurse is using Maslow’s hierarchy to prioritize care for an anxious patient that is not eating and will not see family members. Which area should the nurse address first? a. Anxiety b. Not eating c. Mental health d. Not seeing family members 6. The patient is reporting moderate incisional pain that was not relieved by the last dose of pain medication. The patient is not due for another dose of medication for another 2 1/2 hours. The nurse repositions the patient, asks what type of music the patient likes, and sets the television to the channel playing that type of music. Which health care model is the nurse using? a. Health belief model b. Holistic health model c. Health promotion model d. Maslow’s hierarchy of needs 7. A nurse is assessing internal variables that are affecting the patient’s health status. Which area should the nurse assess? a. Perception of functioning b. Socioeconomic factors c. Cultural background d. Family practices 8. The nurse is admitting a patient with uncontrolled diabetes mellitus. It is the fourth time the patient is being admitted in the last 6 months for high blood sugars. During the admission process, the nurse asks the patient about employment status and displays a nonjudgmental attitude. What is the rationale for the nurse’s actions? a. External variables have little effect on compliance. b. A person’s compliance is affected by economic status. c. Employment status is an internal variable that impacts compliance. d. Noncompliant patients thrive on the disapproval of authority figures. 9. The nurse is working on a committee to evaluate the need for increasing the levels of fluoride in the drinking water of the community. Which concept is the nurse fostering? a. Illness prevention b. Wellness education c. Active health promotion d. Passive health promotion 10. The nurse is working in a clinic that is designed to provide health education and immunizations. Which type of preventive care is the nurse providing? a. Primary prevention b. Secondary prevention c. Tertiary prevention d. Risk factor prevention 11. The patient is admitted to the emergency department of the local hospital from home with reports of chest discomfort and shortness of breath. The patient is placed on oxygen, has labs and blood gases drawn, and is given an electrocardiogram and breathing treatments. Which level of preventive care is this patient receiving? a. Primary prevention b. Secondary prevention c. Tertiary prevention d. Health promotion 12. A patient is admitted to a rehabilitation facility following a stroke. The patient has right-sided paralysis and is unable to speak. The patient will be receiving physical therapy and speech therapy. Which level of preventive care is the patient receiving? a. Primary prevention b. Secondary prevention c. Tertiary prevention d. Health promotion 13. Upon completing a history, the nurse finds that a patient has risk factors for lung disease. How should the nurse interpret this finding? a. A person with the risk factor will get the disease. b. The chances of getting the disease are increased. c. Risk modification will have no effect on disease prevention. d. The disease is guaranteed not to develop if the risk factor is controlled. 14. The nurse is caring for a patient who has been trying to quit smoking. The patient has been smoke free for 2 weeks but had two cigarettes last night and at least two this morning. What should the nurse anticipate? a. The patient does not want to and will never quit smoking. b. The patient must pick up the attempt right where the patient left off. c. The patient will return to the contemplation or precontemplation phase. d. The patient will need to adopt a new lifestyle for change to be effective. 15. The nurse is working in a drug rehabilitation clinic and is in the process of admitting a patient for “detox.” What should the nurse do next? a. Identify the patient’s stage of change. b. Realize that the patient is ready to change. c. Teach the patient that choices will have to change. d. Instruct the patient that relapses will not be tolerated. 16. A female patient has been overweight for most of her life. She has tried dieting in the past and has lost weight, only to regain it when she stopped dieting. The patient is visiting the weight loss clinic/health club because she has decided to do it. She states that she will join right after the holidays, in 3 months. Which stage is the patient displaying? a. Precontemplation b. Contemplation c. Preparation d. Action 17. Upon completion of the assessment, the nurse finds that the patient has quit drinking and has been alcohol free for the past 2 years. Which stage best describes the nurse’s assessment finding? a. Contemplation b. Maintenance c. Preparation d. Action 18. The patient had a colostomy placed 1 week ago. When approached by the nurse, the patient and spouse refuse to talk about it and refuse to be taught about how to care for it. How will the nurse evaluate this couple’s stage of adjustment? a. Shock b. Withdrawal c. Acceptance d. Rehabilitation 19. A patient has had emphysema (lung disease) for many years. When approached by the nurse, the patient states “I would be better off dead.” The patient supports the family, and now because of oxygen dependency the patient must quit work. The patient’s spouse will have to go to work. Which action should the nurse take? a. Develop a plan of care for the family. b. Contact psychiatric services for a referral. c. Assure the patient that things will work out. d. Focus the plan of care solely on maximizing patient function. 20. A nurse is teaching about the transtheoretical model of change. In which order will the nurse place the progression of the stages from beginning to end? 1. Action 2. Preparation 3. Maintenance 4. Contemplation 5. Precontemplation a. 5, 4, 2, 1, 3 b. 2, 5, 4, 3, 1 c. 4, 5, 3, 1, 2 d. 1, 5, 2, 3, 4 MULTIPLE RESPONSE 1. Which areas should the nurse assess to determine the effects of external variables on a patient’s illness? (Select all that apply.) a. Patient’s perception of the illness b. Patient’s coping skills c. Socioeconomic status d. Cultural background e. Social support , D, E 2. A nurse meets the following goals: helps a patient maintain health and helps a patient with an illness. Which factors assist the nurse in achieving these goals? (Select all that apply.) a. Understands the challenges of today’s health care system b. Identifies actual and potential risk factors c. Has coined the term “illness behavior” d. Minimizes the effects of illnesses e. Experiences compassion fatigue , B, D Chapter 07: Caring in Nursing Practice 1. A nurse is caring for a patient in pain. Which nursing approach is priority? a. Relationship-centered b. Technology-centered c. High tech-centered d. Family-centered 2. A nurse is providing pain medication to patients after surgery. Which component is key for the nurse’s personal philosophy of nursing? a. Caring b. Technology c. Informatics d. Therapeutics 3. A nurse attends a seminar on nursing theories for caring. Which information from the nurse indicates a correct understanding of these theories? a. Benner identifies caring as highly connected involving patient and nurse. b. Swanson develops four caring processes to convey caring in nursing. c. Watson’s transcultural caring views inclusion of culture as caring. d. Leininger’s theory places care before cure and is transformative. 4. The patient has a colostomy but has not yet been able to look at it. The nurse teaches the patient how to care for the colostomy. The nurse sits with the patient, and together they form a plan on how to approach dealing with colostomy care. Which caring process is the nurse performing? a. Knowing b. Doing for c. Enabling d. Maintaining belief 5. A nurse is using Watson’s model to provide care to patients. Which carative factor will the nurse use? a. Maintaining belief b. Instilling faith-hope c. Maintaining ethics d. Instilling values 6. A nurse provides care that is receptive to patients’ and families’ perceptions of caring. Which action will the nurse take? a. Provides clear, accurate information b. Just performs nursing tasks competently c. Does as much for the patient as possible d. Focuses solely on the patient’s diagnosis 7. A nurse follows the “ethics of care” when working with patients. Which action will the nurse take? a. Becomes the patient’s advocate based on the patient’s wishes b. Makes decisions for the patient solely using analytical principles c. Uses only intellectual principles to determine what is best for the patient d. Ignores unequal family relationships since that is a personal matter for the family 8. A nurse is providing presence to a patient and the family. Which nursing action does this involve? a. Focusing on the task that needs to be done b. Providing closeness and a sense of caring c. Jumping in to provide patient comfort d. Being there without an identified goal 9. The patient is afraid to have a thoracentesis at the bedside. The nurse sits with the patient and asks about the fears. During the procedure, the nurse stays with the patient, explaining each step and providing encouragement. What is the nurse displaying? a. Providing touch b. Providing a presence c. Providing family care d. Providing a listening ear 10. The patient is terminal and very near death. When the nurse checks the patient and finds no pulse or blood pressure, the family begins sobbing and hugging each other. Some family members hold the patient’s hand. The nurse is overwhelmed by the presence of grief and leaves the room. What is the nurse demonstrating? a. Caring touch b. Protective touch c. Therapeutic touch d. Task-oriented touch 11. Which action indicates a nurse is using caring touch with a patient? a. Inserts a catheter b. Rubs a patient’s back c. Prevents a patient from falling d. Administers an injection 12. The nurse is caring for a patient who has been sullen and quiet for the past three days. Suddenly, the patient says, “I’m really nervous about surgery tomorrow, but I’m more worried about how it will affect my family.” What should the nurse do first? a. Assure the patient that everything will be all right. b. Tell the patient that there is no need to worry. c. Listen to the patient’s concerns and fears. d. Inform the patient a social worker is available. 13. The patient is about to undergo a certain procedure and has voiced concern about outcomes and prognosis. The nurse caring for the patient underwent a similar procedure and stops to listen. Which response by the nurse may be most beneficial? a. “I had a similar procedure and I can tell you what I went through.” b. “I think you’ll be all right, but, of course, there are no sure guarantees.” c. “I don’t think you have anything to worry about. They do lots of these.” d. “I can call the doctor and cancel the procedure, if you are really concerned.” 14. In making rounds, the nurse meets a patient for the first time. The nurse asks the patient when morning medications are taken, such as before breakfast, after breakfast, or during breakfast. What does knowing the patient allow the nurse to do? a. Choose the most appropriate time to give the medication. b. Know what information to put on the medication error report form. c. Explain to the patient that the medication will not be given at the usual time. d. Evaluate whether or not the patient is taking the medication correctly at home. MULTIPLE RESPONSE 1. A nurse cares for patients. Which areas does caring influence? (Select all that apply.) a. The way in which patients feel b. The way in which patients learn c. The way in which patients think d. The way in which patients study e. The way in which patients behave , C, E 2. Which actions by the nurse should be done in order to get to know the patient? (Select all that apply.) a. Avoid assumptions b. Focus on the patient c. Engage in a caring relationship d. Form the relationship very quickly e. Not address spiritual or higher needs , B, C 3. Which actions by the nurse indicate compassion and caring to patients? (Select all that apply.) a. Saying “I’m here” b. Including the family in care c. Staying with the patient during a bedside test d. Relying on monitors and technology e. Refining work processes on the unit , B, C MATCHING Match the examples to the areas the nurse will promote connectedness for patient’s spirituality needs. a. Connection with others b. Connection with higher power c. Connection with oneself 1. Intrapersonally 2. Interpersonally 3. Transpersonally 1.ANS:CDIF:Understand (comprehension)REF:86 OBJ: Discuss the relationship of compassion to caring. TOP: Caring MSC: Psychosocial Integrity 2.ANS:ADIF:Understand (comprehension)REF:86 OBJ: Discuss the relationship of compassion to caring. TOP: Caring MSC: Psychosocial Integrity 3.ANS:BDIF:Understand (comprehension)REF:86 OBJ: Discuss the relationship of compassion to caring. TOP: Caring MSC: Psychosocial Integrity Chapter 09: Cultural Awareness 1. A nurse is working at a health fair screening people for liver cancer. Which population group should the nurse monitor most closely for liver cancer? a. Hispanic b. Asian Americans c. Non-Hispanic Caucasians d. Non-Hispanic African-Americans 2. A nurse is caring for an immigrant with low income. Which information should the nurse consider when planning care for this patient? a. There is a decreased frequency of morbidity. b. There is an increased incidence of disease. c. There is an increased level of health. d. There is a decreased mortality rate. 3. A nurse is assessing the health care disparities among population groups. Which area is the nurse monitoring? a. Accessibility of health care services b. Outcomes of health conditions c. Prevalence of complications d. Incidence of diseases 4. A nurse is providing care to a patient from a different culture. Which action by the nurse indicates cultural competence? a. Communicates effectively in a multicultural context b. Functions effectively in a multicultural context c. Visits a foreign country d. Speaks a different language 5. The nurse learns about cultural issues involved in the patient’s health care belief system and enables patients and families to achieve meaningful and supportive care. Which concept is the nurse demonstrating? a. Marginalized groups b. Health care disparity c. Transcultural nursing d. Culturally congruent care 6. A nurse is beginning to use patient-centered care and cultural competence to improve nursing care. Which step should the nurse take first? a. Assessing own biases and attitude b. Learning about the world view of others c. Understanding organizational forces d. Developing cultural skills 7. A nurse is performing a cultural assessment using the ETHNIC mnemonic for communication. Which area will the nurse assess for the “H”? a. Health b. Healers c. History d. Homeland 8. The nurse is caring for a patient of Hispanic descent who speaks no English. The nurse is working with an interpreter. Which action should the nurse take? a. Use long sentences when talking. b. Look at the patient when talking. c. Use breaks in sentences when talking. d. Look at only nonverbal behaviors when talking. 9. Which action indicates the nurse is meeting a primary goal of cultural competent care for patients? a. Provides care to transgender patients b. Provides care to restore relationships c. Provides care to patients that is individualized d. Provides care to surgical patients 10. The nurse is caring for a Chinese patient using the Teach-Back technique. Which action by the nurse indicates successful implementation of this technique? a. Asks, “Does this make sense?” b. Asks, “Do you think you can do this at home?” c. Asks, “What will you tell your spouse about changing the dressing?” d. Asks, “Would you tell me if you don’t understand something so we can go over it?” 11. A nurse is using core measures to reduce health disparities. Which group should the nurse focus on to cause the most improvement in core measures? a. Caucasians b. Poor people c. Alaska Natives d. American Indians 12. A nurse is designing a form for lesbian, gay, bisexual, and transgender (LGBT) patients. Which design should the nurse use? a. Use partnered rather than married. b. Use mother rather than father. c. Use parents rather than guardian. d. Use wife/husband rather than significant other. 13. A nurse is assessing population groups for the risk of suicide requiring medical attention. Which group should the nurse monitor most closely? a. Young bisexuals b. Young caucasians c. Asian Americans d. African-Americans 14. A nurse is assessing a patient’s ethnohistory. Which question should the nurse ask? a. What language do you speak at home? b. How different is your life here from back home? c. Which caregivers do you seek when you are sick? d. How different is what we do from what your family does when you are sick? 15. A nurse is teaching patients about health care information. Which patient will the nurse assess closely for health literacy? a. A patient 35 years old b. A patient 68 years old c. A patient with a college degree d. A patient with a high-school diploma 16. A nurse works at a hospital that uses equity-focused quality improvement. Which strategy is the hospital using? a. Document staff satisfaction. b. Focus on the family. c. Implement change on a grand scale. d. Reduce disparities. 17. A nurse is providing care to a culturally diverse population. Which action indicates the nurse is successful in the role of providing culturally congruent care? a. Provides care that fits the patient’s valued life patterns and set of meanings b. Provides care that is based on meanings generated by predetermined criteria c. Provides care that makes the nurse the leader in determining what is needed d. Provides care that is the same as the values of the professional health care system 18. A nurse is assessing the patient’s meaning of illness. Which area of focus by the nurse ispriority? a. On the way a patient reacts to disease b. On the malfunctioning of biological processes c. On the malfunctioning of psychological processes d. On the way a patient reacts to family/social interactions MULTIPLE RESPONSE 1. A nurse is using Campinha-Bacote’s model of cultural competency. Which areas will the nurse focus on to become competent? (Select all that apply.) a. Cultural skills b. Cultural desire c. Cultural transition d. Cultural knowledge e. Cultural encounters , B, D, E 2. A nurse is using the RESPECT mnemonic to establish rapport, the “R” in RESPECT. Which actions should the nurse take? (Select all that apply.) a. Connect on a social level. b. Help the patient overcome barriers. c. Consciously attempt to suspend judgment. d. Stress that they will be working together to address problems. e. Know limitations in addressing medical issues across cultures. , C 3. A nurse is using the explanatory model to determine the etiology of an illness. Which questions should the nurse ask? (Select all that apply.) a. How should your sickness be treated? b. What do you call your problem? c. How does this illness work inside your body? d. What do you fear most about your sickness? e. What name does it have? , C, E MATCHING A nurse is using Campinha-Bacote’s model of cultural competency to improve cultural care. Which actions describe the components the nurse is using? a. In-depth self-examination of one’s own background b. Ability to assess factors that influence treatment and care c. Sufficient comparative understanding of diverse groups d. Motivation and commitment to continue learning about cultures e. Cross-cultural interaction that develops communication skills 1. Cultural skills 2. Cultural desires 3. Cultural awareness 4. Cultural knowledge 5. Cultural encounters 1.ANS:BDIF:Understand (comprehension)REF:104-105 OBJ: Describe steps toward developing cultural competence. TOP: Assessment MSC:Management of Care 2.ANS DIF:Understand (comprehension)REF:104-105 OBJ: Describe steps toward developing cultural competence. TOP: Assessment MSC:Management of Care 3.ANS:ADIF:Understand (comprehension)REF:104-105 OBJ: Describe steps toward developing cultural competence. TOP: Assessment MSC:Management of Care 4.ANS:CDIF:Understand (comprehension)REF:104-105 OBJ: Describe steps toward developing cultural competence. TOP: Assessment MSC:Management of Care 5.ANS:EDIF:Understand (comprehension)REF:104-105 OBJ: Describe steps toward developing cultural competence. TOP: Assessment MSC:Management of Care Chapter 10: Caring for Families 1. A nurse is assessing the family unit to determine the family’s ability to adapt to the change of a member having surgery. Which area is the nurse monitoring? a. Family durability b. Family resiliency c. Family diversity d. Family forms 2. A nurse reviews the current trends affecting the family. Which trend will the nurse find? a. Mothers are staying at home. b. Adolescent mothers usually live on their own. c. More grandparents are raising their grandchildren. d. Teenage fathers usually have stronger support systems. 3. A spouse brings the children in to visit their mother in the hospital. The nurse asks how the family is doing. The husband states, “None of her jobs are getting done, and I don’t do those jobs, so the house and the kids are falling apart.” How will the nurse interpret this finding? a. The family structure is resilient. b. The family structure is flexible. c. The family structure is hardy. d. The family structure is rigid. 4. A nurse cares for the family’s as well as the patient’s needs using available resources. Which approach is the nurse using? a. Family as context b. Family as patient c. Family as system d. Family as caregivers 5. A nurse is caring for a patient who needs constant care in the home setting and for whom most of the care is provided by the patient’s family. Which action should the nurse take to help relieve stress? a. Encourage caregiver to do as much as possible. b. Focus primarily on the patient. c. Point out weaknesses. d. Provide education. 6. A nurse is working with a patient. When the nurse asks about family members, the patient states that it includes my spouse, children, and aunt and uncle. How will the nurse describe this type of family? a. Nuclear b. Blended c. Extended d. Alternative 7. A nurse is assessing a child that lives in a car with family members who presents to the emergency department. Which area should the nurse assess closely? a. Ears b. Eyes c. Head d. Hands 8. The nurse is interviewing a patient who is being admitted to the hospital. The patient’s family went home before the nurse’s interview. The nurse asks the patient, “Who decides when to come to the hospital?” What is the rationale for the nurse’s action? a. To assess the family form b. To assess the family function c. To assess the family structure d. To assess the family generalization 9. A nurse is caring for a patient from a motor vehicle accident. Which action by the unlicensed assistive personnel will cause the nurse to intervene? a. Tells the family not to leave the bedside b. Offers the family a sandwich c. Gives the family a blanket d. Sits with the family 10. A nurse is using the family as context approach to provide care to a patient. What should the nurse do next? a. Assess family patterns versus individual characteristics. b. Assess how much the family provides the patient’s basic needs. c. Use “family as patient” and “family as context” approaches simultaneously. d. Plan care to meet not only the patient’s needs but those of the family as well. 11. The nurse is caring for a patient in hospice. The nurse notes that the patient is getting adequate care, but the spouse is not sleeping well. The nurse also assesses the need for better family nutrition and meals assistance. The nurse discusses these needs with the patient and family and develops a plan of care with them using community resources. Which approach is the nurse using? a. Family as context b. Family as patient c. Family as system d. Family as caregiver 12. The nurse is caring for an older adult patient who has no apparent family. When questioned about family and the definition of family, the patient states, “I have no family. They’re all gone.” When asked, “Who prepares your meals?” the patient states, “I do, or I go out.” Which approach should the nurse use for this patient? a. Family as context b. Family as patient c. Family as system d. Family as caregiver 13. The nurse is caring for an older adult patient at home who requires teaching for dressing changes. The spouse and adult child are also involved in changing the dressing. Which statement by the nurse will most likely elicit a positive response from the patient and family? a. “You’re doing that all wrong. Let me show you how to do it.” b. “I don’t know who showed you how to change a dressing, but you’re not doing it right. Let me show you again.” c. “You’re hesitant about changing the dressing like I was before I was shown an easier way; would you like to see?” d. “I used to change the dressing the same way you are doing it: the wrong way. I’ll show you the right way to do it.” 14. The nurse is providing discharge teaching for an older-adult patient who will need tube feedings at home. The spouse is the only source of care and states “I will not be able to perform the feedings due to arthritis.” Which action should the nurse take? a. Obtain extra feeding supplies. b. Arrange for home care. c. Cancel the discharge. d. Teach the spouse. MULTIPLE RESPONSE 1. A nurse is assessing threats concerning the family. Which areas will the nurse include in the assessment? (Select all that apply.) a. Homelessness b. Domestic violence c. Presence of illness d. Changing economic status e. Rise of homosexual families , B, C, D 2. A nurse is assessing the realms of family life. Which processes will the nurse assess? (Select all that apply.) a. Developmental b. Interactive c. Integrity d. Coping e. Life , B, C, D MATCHING A nurse is focusing on the interactive processes of family life and is asking the patient questions. Match the questions the nurse will ask to the interactive process. a. Intimacy expression b. Social support c. Roles d. Family nurturing 1. Who is the “peacekeeper” of the family? 2. How are house rules established? 3. How often does the family hug each other? 4. Who at your workplace is close to the family? 1.ANS:CDIF:Analyze (analysis)REF:124 OBJ: Use the nursing process to provide for the health care needs of the family. TOP: Assessment MSC: Psychosocial Integrity Chapter 11: Developmental Theories 1. When caring for a middle-aged adult exhibiting maladaptive coping skills, the nurse is trying to determine the cause of the patient’s behavior. Which information from a growth and development perspective should the nurse consider when planning care? a. Individuals have uniform patterns of growth and development. b. Culture usually has no effect on predictable patterns of growth and development. c. Health is promoted based on how many developmental failures a patient experiences. d. When individuals experience repeated developmental failures, inadequacies sometimes result. 2. A nurse is measuring an infant’s head circumference and height. Which area is the nurse assessing? a. Moral development b. Cognitive development c. Biophysical development d. Psychosocial development 3. Which question will be most appropriate for a nurse to ask when assessing an adult patient for growth and developmental delays? a. “How many times per week do you exercise?” b. “Are you able to stand on one foot for 5 seconds?” c. “Would you please describe your usual activities during the day?” d. “How many hours a day do you spend watching television or sitting in front of a computer?” 4. A nurse is using the proximodistal pattern to assess an infant’s growth and development as normal. Which assessment finding will the nurse determine as normal? a. Bangs objects before turns b. Lifts head before grasps c. Walks before crawls d. Laughs before coos 5. A nurse is assessing an 18-month-old toddler. The nurse distinguishes normal from abnormal findings by remembering Gesell’s theory of development. Which information will the nurse consider? a. Growth in humans is determined solely by heredity. b. Environmental influence does not influence development. c. The cephalocaudal pattern describes the sequence in which growth is fastest at the top. d. Gene activity influences development but does not affect the sequence of development. 6. A nurse is working with a patient who wants needs to be met and is impatient and demanding when these needs are not met immediately. How should the nurse interpret this finding according to Freud? a. The id is functioning. b. The ego is functioning. c. The superego is functioning. d. The Oedipus complex is functioning. 7. The nurse is teaching a young-adult couple about promoting the health and psychosocial development of their 8-year-old child. Which information from the parent indicates a correct understanding of the teaching? a. “We will provide consistent, devoted relationships to meet needs.” b. “We will limit choices and provide punishment for mistakes.” c. “We will provide proper support for learning new skills.” d. “We will instill a strong identity of who our child is.” 8. A nurse is using Jean Piaget’s developmental theory to focus on cognitive development. Which area will the nurse assess in this patient? a. Latency b. Formal operations c. Intimacy versus isolation d. The postconventional level 9. A nurse is assessing a 17-year-old adolescent’s cognitive development. Which behavior indicates the adolescent has reached formal operations? a. Uses play to understand surroundings b. Discusses the topic of justice in society c. Hits other students to deal with environmental change d. Questions where the ice is hiding when ice has melted in a drink 10. A nurse is caring for a 4-year-old patient. Which object will the nurse allow the child to play with safely to foster cognitive development? a. The pump administering intravenous fluids b. A book to read alone in a quiet place c. The blood pressure cuff d. A baseball bat 11. A patient follows all the instructions a nurse provides because the patient wants to be perceived as a “good” patient. How should the nurse interpret this information according to moral development? a. The patient is in postformal thought reasoning. b. The patient is in postconventional reasoning. c. The patient is in preconventional reasoning. d. The patient is in conventional reasoning. 12. An 18-month-old patient is brought into the clinic for evaluation because the parent is concerned. The 18-month-old child hits siblings and says only “No” when communicating verbally. Which recommendation by the nurse will be best for this situation? a. Assure the mother that the child is developmentally within specified norms. b. Encourage the mother to seek psychological counseling for the child. c. Consult the social worker because the child is hitting other children. d. Remove all toys from the child’s room until this behavior ceases. 13. A formerly independent older adult becomes severely withdrawn upon admission to a nursing home. Which action should the nurse take first? a. Offer a reward to the patient for participation in all events. b. Encourage the patient to eat meals in the dining room with others. c. Allow the patient to incorporate personal belongings into the room. d. Advise the patient of the importance of attending mandatory activities. 14. The nurse is caring for a 14-year-old patient in the hospital. Which goal will be priority? a. Maintain industry b. Maintain identity c. Maintain intimacy d. Maintain initiative 15. The nurse is teaching the parents of a 3-year-old child who is at risk for developmental delays. Which instruction will the nurse include in the teaching plan? a. Insist that your child discuss various points of view. b. Encourage play as your child is exploring the surroundings. c. Discuss world events with your child to foster language development. d. Actively encourage your child to read lengthy books to foster reading abilities. 16. A nurse is caring for a young adult after surgery. Which action by the nurse will be priority? a. Allow involvement of peers b. Allow involvement of partner c. Allow involvement of volunteer activities d. Allow involvement of consistent schedule 17. A nurse takes the history of a middle-aged patient in a health clinic. Which information indicates the patient has achieved generativity? a. Married for 30 years b. Teaches preschoolers c. Has no regrets with life choices d. Cares for aging parents after work 18. Which action should the nurse take when teaching a 5-year-old patient about a scheduled surgery? a. Do not discuss the procedure with the child to decrease anxiety. b. Let the child know the surgery will be at 9:00 AM in the morning. c. Insist that the parents wait outside the room to ensure privacy of the child. d. Allow the child to touch and hold medical equipment such as thermometers. 19. A nurse works on a pediatric unit and is using a psychosocial developmental approach to child care. In which order from the first to the last will the nurse place the developmental stages? 1. Initiative versus guilt 2. Trust versus mistrust 3. Industry versus inferiority 4. Identity versus role confusion 5. Autonomy versus shame and doubt a. 2, 5, 3, 1, 4 b. 2, 1, 3, 5, 4 c. 2, 3, 1, 5, 4 d. 2, 5, 1, 3, 4 MULTIPLE RESPONSE 1. A nurse is developing a plan of care concerning growth and development for a hospitalized adolescent. What should the nurse do? (Select all that apply.) a. Apply developmental theories when making observations of the adolescent’s patterns of growth and development. b. Compare the adolescent’s assessment findings versus normal findings. c. Recognize her own (the nurse’s) moral developmental level. d. Stick with one developmental theory for consistency. e. Apply a unidimensional life span perspective. , B, C 2. A nurse is assessing temperaments of children. Which terms should the nurse use to describe findings? (Select all that apply.) a. The easy child b. The defiant child c. The difficult child d. The slow-to-warm up child e. The momma’s boy or daddy’s girl , C, D Chapter 12: Conception Through Adolescence 1. A mother has delivered a healthy newborn. Which action is priority? a. Encourage close physical contact as soon as possible after birth. b. Isolate the newborn in the nursery during the first hour after delivery. c. Never leave the newborn alone with the mother during the first 8 hours after delivery. d. Do not allow the newborn to remain with parents until the second hour after delivery. 2. A nurse teaches a new mother about the associated health risks to the infant. Which statement by the mother indicates a correct understanding of the teaching? a. “I will feed my baby every 4 hours around-the-clock.” b. “I need to leave the blankets off my baby to prevent smothering.” c. “I need to remind friends who want to hold my baby to wash their hands.” d. “I will throw away the bulb syringe now because my baby is breathing fine.” 3. A nurse is working in the delivery room. Which action is priority immediately after birth? a. Open the airway. b. Determine gestational age. c. Monitor infant-parent interactions. d. Promote parent-newborn physical contact. 4. A nurse is assessing a newborn that was just born. Which newborn finding will cause the nurse to intervene immediately? a. Molding b. A lack of reflexes c. Cyanotic hands and feet d. A soft, protuberant abdomen 5. A nurse performs an assessment on a healthy newborn. Which assessment finding will the nurse document as normal? a. Cyanosis of the feet and hands for the first 48 hours b. Triangle-shaped anterior fontanel c. Sporadic motor movements d. Weight of 4800 grams 6. A nurse is teaching the staff about development. Which information indicates the nurse needs to follow up? a. “Development proceeds in a cephalocaudal pattern.” b. “Development proceeds in a proximal-distal pattern.” c. “Development proceeds at a slower rate during the embryonic stage.” d. “Development proceeds at a predictive rate from the moment of conception.” 7. A nurse is comparing physical growth patterns between school-aged children and adolescents. Which principle should the nurse consider? a. Physical growth usually slows during the adolescent period. b. Secondary sex characteristics usually develop during the adolescent years. c. Boys usually exceed girls in height and weight by the end of the school years. d. The distribution of muscle and fat remains constant during the adolescent years. 8. The parent brings a child to the clinic for a 12-month well visit. The child weighed 6 pounds 2 ounces and was 21 inches long at birth. Which finding will cause the nurse to intervene? a. Height of 30 inches b. Weight of 16 pounds c. Is not yet potty-trained d. Is not yet walking up stairs 9. A nurse is assessing the cognitive changes in a preschooler. Which standard will the nurse use to determine normal? a. The ability to think abstractly and deal effectively with hypothetical problems b. The ability to think in a logical manner about the here and now c. The ability to assume the view of another person d. The ability to classify objects by size or color 10. The nurse is teaching a parenting class. One of the topics is development. Which statement from a parent indicates more teaching is needed? a. “The toddler may use parallel play.” b. “The preschooler has the ability to play in small groups.” c. “The school-aged child still needs total assistance in all safety activities.” d. “The toddler may have temper tantrums from parent’s acting on safety rules.” 11. The nurse is observing a 2-year-old hospitalized patient in the playroom. Which activity will the nurse most likely observe? a. Seeking out same sex children to play with b. Participating as the leader of a small group activity c. Sitting beside another child while playing with blocks d. Separating building blocks into groups by size and color 12. A nurse is communicating with a newly admitted teenaged patient. Which action should the nurse take? a. Avoid questioning the patient about cigarette use when the nurse observes a cigarette lighter lying on the bedside table. b. Complete the admission database as quickly as possible by asking yes and no questions. c. Look for meaning behind the patient’s words and actions. d. Ignore the patient’s withdrawn behavior. 13. A nurse is caring for a preschooler. Which fear should the nurse most plan to minimize? a. Fear of bodily harm b. Fear of weight gain c. Fear of separation d. Fear of strangers 14. A nurse is teaching a class about the effects of nutrition on fetal growth and development. A pregnant patient asks the nurse how much weight should normally be gained over the pregnancy. Which information should the nurse share with the patient? a. About 10 to 20 pounds b. About 15 to 25 pounds c. About 20 to 30 pounds d. About 25 to 35 pounds 15. The nurse is caring for an infant. Which activity is most appropriate for the nurse to offer to the infant? a. Set of cards to organize and separate into groups b. Set of sock puppets with movable eyes c. Set of plastic stacking rings d. Set of paperback book 16. A mother expresses concern because her 5-year-old child frequently talks about friends who don’t exist. What is the nurse’s best response to this mother’s concern? a. “Have you considered a child psychological evaluation?” b. “You should stop your child from playing electronic games.” c. “Pretend play is a sign your child watches too much television.” d. “It’s very normal for a child this age to have imaginary playmates.” 17. A school nurse is encouraging children to play a game of kickball. Which group of children is the nurse most likely addressing? a. Infant b. Toddler c. Preschool d. School-aged 18. Which assessment finding of a school-aged patient should alert the nurse to a possible developmental delay? a. Verbalization of “I have no friends” b. Absence of secondary sex characteristics c. Curiosity about sexuality d. Lack of group identity 19. The nurse is teaching a parent about developmental needs of a 9-month-old infant. Which statement from the parent indicates a correct understanding of the teaching? a. “My child will begin to speak in sentences by 1 year of age.” b. “My child will probably enjoy playing peek-a-boo.” c. “My child will sleep about 7 to 8 hours a night.” d. “My child will be ready to try low-fat milk.” 20. A nurse is teaching the parents of a school-aged child about accidents most common in this age group. Which topic should the nurse address? a. Falls b. Fires c. Drownings d. Poisonings 21. Which information from the parent of an 8-month-old infant will cause the nurse to intervene? a. My baby rides in the front-facing car seat when I go to the grocery store. b. I made sure the slats on the crib were less than 2 inches apart. c. I removed the mobile after my baby could reach it. d. My baby cries every time he sees a new person. 22. The nurse is preparing to teach a group of parents with infants about growth and development. Which information should the nurse include in the teaching session? a. 3-month-old infants will be able to bang objects together. b. 4-month-old infants will be able to sit alone with support. c. 5-month-old infants will be able to creep on hands and knees. d. 6-month-old infants will be able to turn from back to abdomen. 23. Which statement, if made by a parent, will require further instruction from the nurse? a. “I should not be surprised that my teenage son has so many friends.” b. “I get worried because my teenage son thinks he’s indestructible.” c. “I should cover for my 10-year-old son when he makes mistakes until he learns the ropes.” d. “I usually have nutritious snacks available because my 10-year-old son is always hungry right after school.” 24. A nurse is teaching parents about appropriate activities for different age groups. Which toy, if selected by the parent of a 12-month-old infant, will indicate a correct understanding of the teaching? a. Busy box b. Electronic games c. Game requiring two to four people d. Small, plastic alphabet letters and magnets MULTIPLE RESPONSE 1. A nurse is teaching a parenting class for families with adolescents. Which health concerns will the nurse include in the teaching session? (Select all that apply.) a. Suicide b. Eating disorders c. Violence/Homicide d. Sexually transmitted infections e. Gonadotropic hormone stimulation , B, C, D MATCHING A nurse is teaching parents about the fine motor skills of infants to help parents understand development growth and needs. Match the information to the correct age that the nurse should include in the teaching session. a. Can place objects into containers b. Pulls a string to obtain an object c. Can hold a baby bottle d. Holds rattle for short periods e. Uses pincer grasp well 1. 2 to 4 months 2. 4 to 6 months 3. 6 to 8 months 4. 8 to 10 months 5. 10 to 12 months 1.ANS DIF:Understand (comprehension)REF:145 OBJ: Discuss ways in which the nurse is able to help parents meet their child’s developmental needs. TOP: Teaching/Learning MSC: Health Promotion and Maintenance 2.ANS:CDIF:Understand (comprehension)REF:145 OBJ: Discuss ways in which the nurse is able to help parents meet their child’s developmental needs. TOP: Teaching/Learning MSC: Health Promotion and Maintenance 3.ANS:BDIF:Understand (comprehension)REF:145 OBJ: Discuss ways in which the nurse is able to help parents meet their child’s developmental needs. TOP: Teaching/Learning MSC: Health Promotion and Maintenance 4.ANS:EDIF:Understand (comprehension)REF:145 OBJ: Discuss ways in which the nurse is able to help parents meet their child’s developmental needs. TOP: Teaching/Learning MSC: Health Promotion and Maintenance 5.ANS:ADIF:Understand (comprehension)REF:145 OBJ: Discuss ways in which the nurse is able to help parents meet their child’s developmental needs. TOP: Teaching/Learning MSC: Health Promotion and Maintenance Chapter 13: Young and Middle Adults 1. A nurse is caring for a young adult. Which goal is priority? a. Maintain peer relationships. b. Maintain family relationships. c. Maintain parenteral relationships. d. Maintain recreational relationships. 2. The nurse is caring for a hospitalized young-adult male who works as a dishwasher at a local restaurant. He states that he would like to get a better job but has no education. How can the nurse best assist this patient psychosocially? a. By providing information and referrals b. By focusing on the patient’s medical diagnoses c. By telling the patient that he needs to go back to school d. By expecting the patient to be flexible in decision making 3. Which goal is priority when the nurse is caring for a middle-aged adult? a. Maintain immediate family relationships. b. Maintain future generation relationships. c. Maintain personal career relationships. d. Maintain work relationships. 4. A nurse is teaching young adults about health risks. Which statement from a young adult indicates a correct understanding of the teaching? a. “It’s probably safe for me to start smoking. At my age, there’s not enough time for cancer to develop.” b. “My mother had appendicitis so this increases my chance for developing appendicitis.” c. “Controlling the amount of stress in my life may decrease the risk of illness.” d. “I don’t do drugs. I do drink coffee, but caffeine is not a drug.” 5. A nurse is choosing an appropriate topic for a young-adult health fair. Which topic should the nurse include? a. Retirement b. Menopause c. Climacteric factors d. Unplanned pregnancies 6. A nurse is assessing the risk of intimate partner violence (IPV) for patients. Which population should the nurse focus on most for IVP? a. White males b. Pregnant females c. Middle-aged adults d. Nonsubstance abusers 7. A patient states that she is pregnant and concerned because she does not know what to expect, and she wants her husband to play an active part in the birthing process. Which information should the nurse share with the patient? a. Lamaze classes can prepare pregnant women and their partners for what is coming. b. The frequency of sexual intercourse is key to helping the husband feel valued. c. After the birth, the stress of pregnancy will disappear and will be replaced by relief. d. After the baby is born, the wife should accept the extra responsibilities of motherhood. 8. Which information from the nurse indicates a correct understanding of emerging adulthood? a. It is a type of young adulthood. b. It is a type of extended adolescence. c. It is a type of independent exploration. d. It is a type of marriage and parenthood. 9. A nurse is planning care for a 30 year old. Which goal is priority? a. Refine self-perception. b. Master career plans. c. Examine life goals. d. Achieve intimacy. 10. A nurse is planning care for young-adult patients. Which information should the nurse consider when planning care? a. Fertility issues do not occur in young adulthood. b. Young adults tend to suffer more from severe illness. c. Substance abuse is easy to observe in young-adult patients. d. Young adults are quite active but are at risk for illness in later years. 11. During a routine physical assessment, the nurse obtaining a health history notes that a 50-year-old female patient reports pain and redness in the right breast. Which action is best for the nurse to take in response to this finding? a. Assess the patient as thoroughly as possible. b. Explain to the patient that breast tenderness is normal at her age. c. Tell the patient that redness is not a cause for concern and is quite common. d. Inform her that redness is the precursor to normal unilateral breast enlargement. 12. A 55-year-old female presents to the outpatient clinic describing irregular menstrual periods and hot flashes. Which information should the nurse share with the patient? a. The patient’s assessment points toward normal menopause. b. Those symptoms are normal when a woman undergoes the climacteric. c. An assessment is not really needed because these problems are normal for older women. d. The patient should stop regular exercise because that is probably causing these symptoms. 13. The nurse is teaching a class to pregnant women about common physiological changes during pregnancy. Which information should the nurse include in the teaching session? a. Pregnancy is not a time to be having sexual activity. b. Urinary frequency will occur early in the pregnancy. c. Breast tenderness should be reported as soon as possible. d. Late in the pregnancy Braxton Hicks contraction may occur. 14. A nurse discusses the risks of repeated sun exposure with a young-adult patient. Which response will the nurse most expect from this patient? a. “I should consider participating in a health fair about safe sun practices.” b. “I’ll make an appointment with my doctor right away for a full skin check.” c. “I’ve had this mole my whole life. So what if it changed color? My skin is fine.” d. “I have a mole that has been bothering me. I’ll call my family doctor for an appointment to get it checked.” 15. Upon assessment of a middle-aged adult, the nurse observes uneven weight bearing and decreased range of joint motion. Which area is priority? a. Abuse potential b. Fall precautions c. Stroke prevention d. Self-esteem issues 16. A young-adult patient is brought to the hospital by police after crashing the car in a high-speed chase when trying to avoid arrest for spousal abuse. Which action should the nurse take? a. Question the patient about drug use. b. Offer the patient a cup of coffee to calm nerves. c. Discretely assess the patient for sexually transmitted infections. d. Deal with the issue at hand, not asking about previous illnesses. 17. A nurse determines that a middle-aged patient is a typical example of the “sandwich generation.” What did the nurse discover the patient is caught between? a. Job responsibilities or family responsibilities b. Stopping old habits and starting new ones c. Caring for children and aging parents d. Advancing in career or retiring MULTIPLE RESPONSE 1. A nurse is assessing a middle-aged patient’s barriers to change in eating habits. Which areas will the nurse assess that are external barriers? (Select all that apply.) a. Lack of facilities b. Lack of materials c. Lack of knowledge d. Lack of social supports e. Lack of short- and long-term goals , B, D 2. A home health nurse is providing care to a middle-aged couple with children at home. The patient has a debilitating chronic illness. Which areas will the nurse need to assess? (Select all that apply.) a. Adherence to treatment and rehabilitation regimens b. Coping mechanisms of patient and family c. Need for community services or referrals d. Knowledge base of patient only e. Use of a doula for care , B, C 3. A nurse is providing prenatal care to a first-time mother. Which information will the nurse share with the patient? (Select all that apply.) a. Regular trend for postpartum depression b. Protection against urinary infection c. Strategies for empty nest syndrome d. Exercise patterns e. Proper diet , D, E MATCHING A nurse is assessing young and middle-aged adults for work-related conditions. Match the job to the work-related conditions that the nurse is assessing. a. Liver disease b. Carpal tunnel syndrome c. Asbestosis d. Farmer’s lung e. Bladder cancer 1. Insulators 2. Dry cleaners 3. Dye workers 4. Office computer workers 5. Agricultural workers 1.ANS:CDIF:Understand (comprehension)REF:164 OBJ: Describe health concerns of the young adult, the childbearing family, and the middle adult. TOP:AssessmentMSC:Health Promotion and Maintenance 2.ANS:ADIF:Understand (comprehension)REF:164 OBJ: Describe health concerns of the young adult, the childbearing family, and the middle adult. TOP:AssessmentMSC:Health Promotion and Maintenance 3.ANS:EDIF:Understand (comprehension)REF:164 OBJ: Describe health concerns of the young adult, the childbearing family, and the middle adult. TOP:AssessmentMSC:Health Promotion and Maintenance 4.ANS:BDIF:Understand (comprehension)REF:164 OBJ: Describe health concerns of the young adult, the childbearing family, and the middle adult. TOP:AssessmentMSC:Health Promotion and Maintenance 5.ANS DIF:Understand (comprehension)REF:164 OBJ: Describe health concerns of the young adult, the childbearing family, and the middle adult. TOP:AssessmentMSC:Health Promotion and Maintenance Chapter 14: Older Adult 1. A nurse is obtaining a history on an older adult. Which finding will the nurse most typically find? a. Lives in a nursing home b. Lives with a spouse c. Lives divorced d. Lives alone 2. A nurse is developing a plan of care for an older adult. Which information will the nurse consider? a. Should be standardized because most geriatric patients have the same needs b. Needs to be individualized to the patient’s unique needs c. Focuses on the disabilities that all aging persons face d. Must be based on chronological age alone 3. Which information from a co-worker on a gerontological unit will cause the nurse to intervene? a. Most older people have dependent functioning. b. Most older people have strengths we should focus on. c. Most older people should be involved in care decision. d. Most older people should be encouraged to have independence. 4. A nurse suspects an older-adult patient is experiencing caregiver neglect. Which assessment findings are consistent with the nurse’s suspicions? a. Flea bites and lice infestation b. Left at a grocery store c. Refuses to take a bath d. Cuts and bruises 5. A nurse is teaching a group of older-adult patients. Which teaching strategy is best for the nurse to use? a. Provide several topics of discussion at once to promote independence and making choices. b. Avoid uncomfortable silences after questions by helping patients complete their statements. c. Ask patients to recall past experiences that correspond with their interests. d. Speak in a high pitch to help patients hear better. 6. An older patient has fallen and suffered a hip fracture. As a consequence, the patient’s family is concerned about the patient’s ability to care for self, especially during this convalescence. What should the nurse do? a. Stress that older patients usually ask for help when needed. b. Inform the family that placement in a nursing center is a permanent solution. c. Tell the family to enroll the patient in a ceramics class to maintain quality of life. d. Provide information and answer questions as family members make choices among care options. 7. What is the best suggestion a nurse could make to a family requesting help in selecting a local nursing center? a. Have the family members evaluate nursing home staff according to their ability to get tasks done efficiently and safely. b. Make sure that nursing home staff members get patients out of bed and dressed according to staff’s preferences. c. Explain that it is important for the family to visit the center and inspect it personally. d. Suggest a nursing center that has standards as close to hospital standards as possible. 8. A 70-year-old patient who suffers from worsening dementia is no longer able to live alone. The nurse is discussing health care services and possible long-term living arrangements with the patient’s only son. What will the nurse suggest? a. An apartment setting with neighbors close by b. Having the patient utilize weekly home health visits c. A nursing center because home care is no longer safe d. That placement is irrelevant because the patient is retreating to a place of inactivity 9. A nurse is caring for an older adult. Which goal is priority? a. Adjusting to career b. Adjusting to divorce c. Adjusting to retirement d. Adjusting to grandchildren 10. A nurse is observing for the universal loss in an older-adult patient. What is the nurse assessing? a. Loss of finances through changes in income b. Loss of relationships through death c. Loss of career through retirement d. Loss of home through relocation 11. A nurse is discussing sexuality with an older adult. Which action will the nurse take? a. Ask closed-ended questions about specific symptoms the patient may experience. b. Provide information about the prevention of sexually transmitted infections. c. Discuss the issues of sexuality in a group in a private room. d. Explain that sexuality is not necessary as one ages. 12. A nurse is teaching a health promotion class for older adults. In which order will the nurse list the most common to least common conditions that can lead to death in older adults? 1. Chronic obstructive lung disease 2. Cerebrovascular accidents 3. Heart disease 4. Cancer a. 4, 1, 2, 3 b. 3, 4, 1, 2 c. 2, 3, 4, 1 d. 1, 2, 3, 4 13. A nurse is observing skin integrity of an older adult. Which finding will the nurse document as a normal finding? a. Oily skin b. Faster nail growth c. Decreased elasticity d. Increased facial hair in men 14. An older-adult patient in no acute distress reports being less able to taste and smell. What is the nurse’s best response to this information? a. Notify the health care provider immediately to rule out cranial nerve damage. b. Schedule the patient for an appointment at a smell and taste disorders clinic. c. Perform testing on the vestibulocochlear nerve and a hearing test. d. Explain to the patient that diminished senses are normal findings. 15. A nurse is assessing an older adult for cognitive changes. Which symptom will the nurse report as normal? a. Disorientation b. Poor judgment c. Slower reaction time d. Loss of language skills 16. An older patient with dementia and confusion is admitted to the nursing unit after hip replacement surgery. Which action will the nurse include in the plan of care? a. Keep a routine. b. Continue to reorient. c. Allow several choices. d. Socially isolate patient. 17. A nurse is helping an older-adult patient with instrumental activities of daily living. The nurse will be assisting the patient with which activity? a. Taking a bath b. Getting dressed c. Making a phone call d. Going to the bathroom 18. A male older-adult patient expresses concern and anxiety about decreased penile firmness during an erection. What is the nurse’s best response? a. Tell the patient that libido will always decrease, as well as the sexual desires. b. Tell the patient that touching should be avoided unless intercourse is planned. c. Tell the patient that heterosexuality will help maintain stronger libido. d. Tell the patient that this change is expected in aging adults. 19. A patient asks the nurse what the term polypharmacy means. Which information should the nurse share with the patient? a. This is multiple side effects experienced when taking medications. b. This is many adverse drug effects reported to the pharmacy. c. This is the multiple risks of medication effects due to aging. d. This is concurrent use of many medications. 20. An outcome for an older-adult patient living alone is to be free from falls. Which statement indicates the patient correctly understands the teaching on safety concerns? a. “I’ll take my time getting up from the bed or chair.” b. “I should dim the lighting outside to decrease the glare in my eyes.” c. “I’ll leave my throw rugs in place so that my feet won’t touch the cold tile.” d. “I should wear my favorite smooth bottom socks to protect my feet when walking around.” 21. A nurse’s goal for an older adult is to reduce the risk of adverse medication effects. Which action will the nurse take? a. Review the patient’s list of medications at each visit. b. Teach that polypharmacy is to be avoided at all cost. c. Avoid information about adverse effects. d. Focus only on prescribed medications. 22. An older-adult patient has developed acute confusion. The patient has been on tranquilizers for the past week. The patient’s vital signs are normal. What should the nurse do? a. Take into account age-related changes in body systems that affect pharmacokinetic activity. b. Increase the dose of tranquilizer if the cause of the confusion is an infection. c. Note when the confusion occurs and medicate before that time. d. Restrict phone calls to prevent further confusion. 23. Which assessment finding of an older adult, who has a urinary tract infection, requires an immediate nursing intervention? a. Confusion b. Presbycusis c. Temperature of 97.9° F d. Death of a spouse 2 months ago 24. Which patient statement is the most reliable indicator that an older adult has the correct understanding of health promotion activities? a. “I need to increase my fat intake and limit protein.” b. “I still keep my dentist appointments even though I have partials now.” c. “I should discontinue my fitness club membership for safety reasons.” d. “I’m up-to-date on my immunizations, but at my age, I don’t need the influenza vaccine.” 25. A 72-year-old woman was recently widowed. She worked as a teller at a bank for 40 years and has been retired for the past 5 years. She never learned how to drive. She lives in a rural area that does not have public transportation. Which psychosocial change does the nurse focus on as a priority? a. Sexuality b. Retirement c. Environment d. Social isolation MULTIPLE RESPONSE 1. A recently widowed older-adult patient is dehydrated and is admitted to the hospital for intravenous fluid replacement. During the evening shift, the patient becomes acutely confused. Which possible reversible causes will the nurse consider when assessing this patient? (Select all that apply.) a. Electrolyte imbalance b. Sensory deprivation c. Hypoglycemia d. Drug effects e. Dementia , B, C, D MATCHING A nurse is using different strategies to meet older patients’ psychosocial needs. Match the strategy the nurse is using to its description. a. Respecting the older adult’s uniqueness b. Improving level of awareness c. Listening to the patient’s past recollections d. Accepting describing of patient’s perspective e. Offering help with grooming and hygiene 1. Body image 2. Validation therapy 3. Therapeutic communication 4. Reality orientation 5. Reminiscence 1.ANS:EDIF:Understand (comprehension)REF:189-190 OBJ: Identify nursing interventions related to the physiological, cognitive, and psychosocial changes of aging. TOP: Implementation MSC: Psychosocial Integrity 2.ANS DIF:Understand (comprehension)REF:189-190 OBJ: Identify nursing interventions related to the physiological, cognitive, and psychosocial changes of aging. TOP: Implementation MSC: Psychosocial Integrity 3.ANS:ADIF:Understand (comprehension)REF:189-190 OBJ: Identify nursing interventions related to the physiological, cognitive, and psychosocial changes of aging. TOP: Implementation MSC: Psychosocial Integrity 4.ANS:BDIF:Understand (comprehension)REF:189-190 OBJ: Identify nursing interventions related to the physiological, cognitive, and psychosocial changes of aging. TOP: Implementation MSC: Psychosocial Integrity 5.ANS:CDIF:Understand (comprehension)REF:189-190 OBJ: Identify nursing interventions related to the physiological, cognitive, and psychosocial changes of aging. TOP: Implementation MSC: Psychosocial Integrity Chapter 15: Critical Thinking in Nursing Practice 1. Which action should the nurse take when using critical thinking to make clinical decisions? a. Make decisions based on intuition. b. Accept one established way to provide care. c. Consider what is important in a given situation. d. Read and follow the heath care provider’s orders. 2. Which patient scenario of a surgical patient in pain is most indicative of critical thinking? a. Administering pain-relief medication according to what was given last shift b. Offering pain-relief medication based on the health care provider’s orders c. Asking the patient what pain-relief methods, pharmacological and nonpharmacological, have worked in the past d. Explaining to the patient that self-reporting of severe pain is not consistent with the minor procedure that was performed 3. Which action indicates a registered nurse is being responsible for making clinical decisions? a. Applies clear textbook solutions to patients’ problems b. Takes immediate action when a patient’s condition worsens c. Uses only traditional methods of providing care to patients d. Formulates standardized care plans solely for groups of patients 4. A charge nurse is supervising the care of a new nurse. Which action by a new nurse indicates the charge nurse needs to intervene? a. Making an ethical clinical decision b. Making an informed clinical decision c. Making a clinical decision in the patient’s best interest d. Making a clinical decision based on previous shift assessments 5. Which action demonstrates a nurse utilizing reflection to improve clinical decision making? a. Obtains data in an orderly fashion b. Uses an objective approach in patient situations c. Improves a plan of care while thinking back on interventions effectiveness d. Provides evidence-based explanations and research for care of assigned patients 6. A nursing instructor needs to evaluate students’ abilities to synthesize data and identify relationships between nursing diagnoses. Which learning assignment is best suited for this instructor’s needs? a. Concept mapping b. Reflective journaling c. Lecture and discussion d. Reading assignment with a written summary 7. A nurse is using a critical thinking model to provide care. Which component is first that helps a nurse make clinical decisions? a. Attitude b. Experience c. Nursing process d. Specific knowledge base 8. Which action by a nurse indicates application of the critical thinking model to make the bestclinical decisions? a. Drawing on past clinical experiences to formulate standardized care plans b. Relying on recall of information from past lectures and textbooks c. Depending on the charge nurse to determine priorities of care d. Using the nursing process 9. A nurse is using the critical thinking skill of evaluation. Which action will the nurse take? a. Examine the meaning of data. b. Support findings and conclusions. c. Review the effectiveness of nursing actions. d. Search for links between the data and the nurse’s assumptions. 10. The patient appears to be in no apparent distress, but vital signs taken by assistive personnel reveal an extremely low pulse. The nurse then auscultates an apical pulse and asks the patient whether there is any history of heart problems. The nurse is utilizing which critical thinking skill? a. Evaluation b. Explanation c. Interpretation d. Self-regulation 11. A patient continues to report postsurgical incision pain at a level of 9 out of 10 after pain medicine is given. The next dose of pain medicine is not due for another hour. What should the critically thinking nurse do first? a. Explore other options for pain relief. b. Discuss the surgical procedure and reason for the pain. c. Explain to the patient that nothing else has been ordered. d. Offer to notify the health care provider after morning rounds are completed. 12. Which action should the nurse take to best develop critical thinking skills? a. Study 3 hours more each night. b. Attend all inservice opportunities. c. Actively participate in clinical experiences. d. Interview staff nurses about their nursing experiences. 13. While caring for a hospitalized older-adult female post hip surgery, the nurse is faced with the task of inserting an indwelling urinary catheter, which involves rotating the hip into a contraindicated position. Which action should the nurse take? a. Postpone catheter insertion until the next shift. b. Adapt the positioning technique to the situation. c. Notify the health care provider for a urologist consult. d. Follow textbook procedure with contraindicated position. 14. The nurse enters a room to find the patient sitting up in bed crying. How will the nurse display a critical thinking attitude in this situation? a. Provide privacy and check on the patient 30 minutes later. b. Set a box of tissues at the patient’s bedside before leaving the room. c. Limit visitors while the patient is upset. d. Ask the patient about the crying. 15. A patient is having trouble reaching the water fountain while holding on to crutches. The nurse suggests that the patient place the crutches against the wall while stabilizing him or herself with two hands on the water fountain. Which critical thinking attitude did the nurse use in this situation? a. Humility b. Creativity c. Risk taking d. Confidence 16. A nurse is pulled from the surgical unit to work on the oncology unit. Which action by the nurse displays humility and responsibility? a. Refusing the assignment b. Asking for an orientation to the unit c. Admitting lack of knowledge and going home d. Assuming that patient care will be the same as on the other units 17. A nurse is using professional standards to influence clinical decisions. What is the rationale for the nurse’s actions? a. Establishes minimal passing standards for testing b. Utilizes evidence-based practice based on nurses’ needs c. Bypasses the patient’s feelings to promote ethical standards d. Uses critical thinking for the highest level of quality nursing care 18. A nurse who is caring for a patient with a pressure ulcer applies the recommended dressing according to hospital policy. Which standard is the nurse following? a. Fairness b. Intellectual standards c. Independent reasoning d. Institutional practice guidelines 19. A nurse is reviewing care plans. Which finding, if identified in a plan of care, should the registered nurse revise? a. Patient’s outcomes for learning b. Nurse’s assumptions about hospital discharge c. Identification of several actual health problems d. Documentation of patient’s ability to meet the goal 20. In which order will the nurse use the nursing process steps during the clinical decision-making process? 1. Evaluating goals 2. Assessing patient needs 3. Planning priorities of care 4. Determining nursing diagnoses 5. Implementing nursing interventions a. 2, 4, 3, 5, 1 b. 4, 3, 2, 1, 5 c. 1, 2, 4, 5, 3 d. 5, 1, 2, 3, 4 MULTIPLE RESPONSE 1. Which findings will alert the nurse that stress is present when making a clinical decision? (Select all that apply.) a. Tense muscles b. Reactive responses c. Trouble concentrating d. Very tired feelings e. Managed emotions , B, C, D Learn to recognize when you are feeling stressed—your muscles will tense, you become reactive when others communicate with you, you have trouble concentrating, and you feel very tired. Emotions are not managed when stressed. DIF:Understand (comprehension)REF:206 OBJ: Discuss the importance of managing stress when making clinical decisions. TOP:AssessmentMSC:Health Promotion and Maintenance Chapter 16: Nursing Assessment 1. The nurse is using critical thinking skills during the first phase of the nursing process. Which action indicates the nurse is in the first phase? a. Completes a comprehensive database b. Identifies pertinent nursing diagnoses c. Intervenes based on priorities of patient care d. Determines whether outcomes have been achieved 2. A nurse is using the problem-oriented approach to data collection. Which action will the nurse take first? a. Complete the questions in chronological order. b. Focus on the patient’s presenting situation. c. Make accurate interpretations of the data. d. Conduct an observational overview. 3. After reviewing the database, the nurse discovers that the patient’s vital signs have not been recorded by the nursing assistive personnel (NAP). Which clinical decision should the nurse make? a. Administer scheduled medications assuming that the NAP would have reported abnormal vital signs. b. Have the patient transported to the radiology department for a scheduled x-ray, and review vital signs upon return. c. Ask the NAP to record the patient’s vital signs before administering medications. d. Omit the vital signs because the patient is presently in no distress. 4. The nurse is gathering data on a patient. Which data will the nurse report as objective data? a. States “doesn’t feel good” b. Reports a headache c. Respirations 16 d. Nauseated 5. A patient expresses fear of going home and being alone. Vital signs are stable and the incision is nearly completely healed. What can the nurse infer from the subjective data? a. The patient can now perform the dressing changes without help. b. The patient can begin retaking all of the previous medications. c. The patient is apprehensive about discharge. d. The patient’s surgery was not successful. 6. Which method of data collection will the nurse use to establish a patient’s database? a. Reviewing the current literature to determine evidence-based nursing actions b. Checking orders for diagnostic and laboratory tests c. Performing a physical examination d. Ordering medications 7. A nurse is gathering information about a patient’s habits and lifestyle patterns. Which method of data collection will the nurse use that will best obtain this information? a. Carefully review lab results. b. Conduct the physical assessment. c. Perform a thorough nursing health history. d. Prolong the termination phase of the interview. 8. While interviewing an older female patient of Asian descent, the nurse notices that the patient looks at the ground when answering questions. What should the nurse do? a. Consider cultural differences during this assessment. b. Ask the patient to make eye contact to determine her affect. c. Continue with the interview and document that the patient is depressed. d. Notify the health care provider to recommend a psychological evaluation. 9. A nurse has already set the agenda during a patient-centered interview. What will the nurse do next? a. Begin with introductions. b. Ask about the chief concerns or problems. c. Explain that the interview will be over in a few minutes. d. Tell the patient “I will be back to administer medications in 1 hour.” 10. The nurse is attempting to prompt the patient to elaborate on the reports of daytime fatigue. Which question should the nurse ask? a. “Is there anything that you are stressed about right now that I should know?” b. “What reasons do you think are contributing to your fatigue?” c. “What are your normal work hours?” d. “Are you sleeping 8 hours a night?” 11. A nurse is conducting a nursing health history. Which component will the nurse address? a. Nurse’s concerns b. Patient expectations c. Current treatment orders d. Nurse’s goals for the patient 12. While the patient’s lower extremity, which is in a cast, is assessed, the patient tells the nurse about an inability to rest at night. The nurse disregards this information, thinking that no correlation has been noted between having a leg cast and developing restless sleep. Which action would have been best for the nurse to take? a. Tell the patient to just focus on the leg and cast right now. b. Document the sleep patterns and information in the patient’s chart. c. Explain that a more thorough assessment will be needed next shift. d. Ask the patient about usual sleep patterns and the onset of having difficulty resting. 13. The nurse begins a shift assessment by examining a surgical dressing that is saturated with serosanguineous drainage on a patient who had open abdominal surgery yesterday (or 1 day ago). Which type of assessment approach is the nurse using? a. Gordon’s Functional Health Patterns b. Activity-exercise pattern assessment c. General to specific assessment d. Problem-oriented assessment 14. Which statement by a nurse indicates a good understanding about the differences between data validation and data interpretation? a. “Data interpretation occurs before data validation.” b. “Validation involves looking for patterns in professional standards.” c. “Validation involves comparing data with other sources for accuracy.” d. “Data interpretation involves discovering patterns in professional standards.” 15. Which scenario best illustrates the nurse using data validation when making a nursing clinical decision for a patient? a. The nurse determines to remove a wound dressing when the patient reveals the time of the last dressing change and notices old and new drainage. b. The nurse administers pain medicine due at 1700 at 1600 because the patient reports increased pain and the family wants something done. c. The nurse immediately asks the health care provider for an order of potassium when a patient reports leg cramps. d. The nurse elevates a leg cast when the patient reports decreased mobility. 16. While completing an admission database, the nurse is interviewing a patient who states “I am allergic to latex.” Which action will the nurse take first? a. Immediately place the patient in isolation. b. Ask the patient to describe the type of reaction. c. Proceed to the termination phase of the interview. d. Document the latex allergy on the medication administration record. 17. A patient verbalizes a low pain level of 2 out of 10 but exhibits extreme facial grimacing while moving around in bed. What is the nurse’s initial action in response to these observations? a. Proceed to the next patient’s room to make rounds. b. Determine the patient does not want any pain medicine. c. Ask the patient about the facial grimacing with movement. d. Administer the pain medication ordered for moderate to severe pain. 18. The nurse is interviewing a patient with a hearing deficit. Which area should the nurse use to conduct this interview? a. The patient’s room with the door closed b. The waiting area with the television turned off c. The patient’s room before administration of pain medication d. The waiting room while the occupational therapist is working on leg exercises 19. A new nurse is completing an assessment on an 80-year-old patient who is alert and oriented. The patient’s daughter is present in the room. Which action by the nurse will require follow-up by the charge nurse? a. The nurse makes eye contact with the patient. b. The nurse speaks only to the patient’s daughter. c. The nurse leans forward while talking with the patient. d. The nurse nods periodically while the patient is speaking. MULTIPLE RESPONSE 1. A nurse is completing an assessment. Which findings will the nurse report as subjective data? (Select all that apply.) a. Patient’s temperature b. Patient’s wound appearance c. Patient describing excitement about discharge d. Patient pacing the floor while awaiting test results e. Patient’s expression of fear regarding upcoming surgery , E MATCHING A nurse is completing an assessment using the PQRST to obtain data about the patient’s chest pain. Match the questions to the components of the PQRST that the nurse will be using. a. Where is the pain located? b. What causes the pain? c. Does it come and go? d. What does the pain feel like? e. What is the rating on a scale of 0 to 10? 1. Provokes 2. Quality 3. Radiate 4. Severity 5.Time 1.ANS:BDIF:Apply (application)REF:219 OBJ: Describe the methods of data collection. TOP: Assessment MSC: Basic Care and Comfort 2.ANS DIF:Apply (application)REF:219 OBJ: Describe the methods of data collection. TOP: Assessment MSC: Basic Care and Comfort 3.ANS:ADIF:Apply (application)REF:219 OBJ: Describe the methods of data collection. TOP: Assessment MSC: Basic Care and Comfort 4.ANS:EDIF:Apply (application)REF:219 OBJ: Describe the methods of data collection. TOP: Assessment MSC: Basic Care and Comfort 5.ANS:CDIF:Apply (application)REF:219 OBJ: Describe the methods of data collection. TOP: Assessment MSC: Basic Care and Comfort Chapter 17: Nursing Diagnosis 1. After assessing a patient, a nurse develops a standard formal nursing diagnosis. What is the rationale for the nurse’s actions? a. To form a language that can be encoded only by nurses b. To distinguish the nurse’s role from the physician’s role c. To develop clinical judgment based on other’s intuition d. To help nurses focus on the scope of medical practice 2. Which diagnosis will the nurse document in a patient’s care plan that is NANDA-I approved? a. Sore throat b. Acute pain c. Sleep apnea d. Heart failure 3. A nurse develops a nursing diagnostic statement for a patient with a medical diagnosis of pneumonia with chest x-ray results of lower lobe infiltrates. Which nursing diagnosis did the nurse write? a. Ineffective breathing pattern related to pneumonia b. Risk for infection related to chest x-ray procedure c. Risk for deficient fluid volume related to dehydration d. Impaired gas exchange related to alveolar-capillary membrane changes 4. The nurse is reviewing a patient’s plan of care, which includes the nursing diagnostic statement, Impaired physical mobility related to tibial fracture as evidenced by patient’s inability to ambulate. Which part of the diagnostic statement does the nurse need to revise? a. Etiology b. Nursing diagnosis c. Collaborative problem d. Defining characteristic 5. A nurse is using assessment data gathered about a patient and combining critical thinking to develop a nursing diagnosis. What is the nurse doing? a. Assigning clinical cues b. Defining characteristics c. Diagnostic reasoning d. Diagnostic labeling 6. A patient presents to the emergency department following a motor vehicle crash and suffers a right femur fracture. The leg is stabilized in a full leg cast. Otherwise, the patient has no other major injuries, is in good health, and reports only moderate discomfort. Which is the mostpertinent nursing diagnosis the nurse will include in the plan of care? a. Posttrauma syndrome b. Constipation c. Acute pain d. Anxiety 7. The nurse is reviewing a patient’s database for significant changes and discovers that the patient has not voided in over 8 hours. The patient’s kidney function lab results are abnormal, and the patient’s oral intake has significantly decreased since previous shifts. Which step of the nursing process should the nurse proceed to after this review? a. Diagnosis b. Planning c. Implementation d. Evaluation 8. A patient with a spinal cord injury is seeking to enhance urinary elimination abilities by learning self-catheterization versus assisted catheterization by home health nurses and family members. The nurse adds Readiness for enhanced urinary elimination in the care plan. Which type of diagnosis did the nurse write? a. Risk b. Problem focused c. Health promotion d. Collaborative problem 9. A nurse administers an antihypertensive medication to a patient at the scheduled time of 0900. The nursing assistive personnel (NAP) then reports to the nurse that the patient’s blood pressure was low when it was taken at 0830. The NAP states that was busy and had not had a chance to tell the nurse yet. The patient begins to complain of feeling dizzy and light-headed. The blood pressure is rechecked and it has dropped even lower. In which phase of the nursing process did the nurse first make an error? a. Assessment b. Diagnosis c. Implementation d. Evaluation 10. A nurse adds the following diagnosis to a patient’s care plan: Constipation related to decreased gastrointestinal motility secondary to pain medication administration as evidenced by the patient reporting no bowel movement in seven days, abdominal distention, and abdominal pain.Which element did the nurse write as the defining characteristic? a. Decreased gastrointestinal motility b. Pain medication c. Abdominal distention d. Constipation 11. The patient database reveals that a patient has decreased oral intake, decreased oxygen saturation when ambulating, reports of shortness of breath when getting out of bed, and a productive cough. Which elements will the nurse identify as defining characteristics for the diagnostic label of Activity intolerance? a. Decreased oral intake and decreased oxygen saturation when ambulating b. Decreased oxygen saturation when ambulating and reports of shortness of breath when getting out of bed c. Reports of shortness of breath when getting out of bed and a productive cough d. Productive cough and decreased oral intake 12. A nurse performs an assessment on a patient. Which assessment data will the nurse use as an etiology for Acute pain? a. Discomfort while changing position b. Reports pain as a 7 on a 0 to 10 scale c. Disruption of tissue integrity d. Dull headache 13. A new nurse writes the following nursing diagnoses on a patient’s care plan. Which nursing diagnosis will cause the nurse manager to intervene? a. Wandering b. Hemorrhage c. Urinary retention d. Impaired swallowing 14. A patient has a bacterial infection in left lower leg. Which nursing diagnosis will the nurse add to the patient’s care plan? a. Infection b. Risk for infection c. Impaired skin integrity d. Staphylococcal leg infection 15. A nurse adds a nursing diagnosis to a patient’s care plan. Which information did the nurse document? a. Decreased cardiac output related to altered myocardial contractility. b. Patient needs a low-fat diet related to inadequate heart perfusion. c. Offer a low-fat diet because of heart problems. d. Acute heart pain related to discomfort. 16. A charge nurse is evaluating a new nurse’s plan of care. Which finding will cause the charge nurse to follow up? a. Assigning a documented nursing diagnosis of Risk for infection for a patient on intravenous (IV) antibiotics b. Completing an interview and physical examination before adding a nursing diagnosis c. Developing nursing diagnoses before completing the database d. Including cultural and religious preferences in the database 17. A patient exhibits the following symptoms: tachycardia, increased thirst, headache, decreased urine output, and increased body temperature. The nurse analyzes the data. Which nursing diagnosis will the nurse assign to the patient? a. Adult failure to thrive b. Hypothermia c. Deficient fluid volume d. Nausea 18. Which question would be most appropriate for a nurse to ask a patient to assist in establishing a nursing diagnosis of Diarrhea? a. “What types of foods do you think caused your upset stomach?” b. “How many bowel movements a day have you had?” c. “Are you able to get to the bathroom in time?” d. “What medications are you currently taking?” 19. A nurse assesses that a patient has not voided in 6 hours. Which question should the nurse ask to assist in establishing a nursing diagnosis of Urinary retention? a. “Do you feel like you need to go to the bathroom?” b. “Are you able to walk to the bathroom by yourself?” c. “When was the last time you took your medicine?” d. “Do you have a safety rail in your bathroom at home?” 20. A nurse is developing nursing diagnoses for a patient. Beginning with the first step, place in order the steps the nurse will use. 1. Observes the patient having dyspnea (shortness of breath) and a diagnosis of asthma 2. Writes a diagnostic label of impaired gas exchange 3. Organizes data into meaningful clusters 4. Interprets information from patient 5. Writes an etiology a. 1, 3, 4, 2, 5 b. 1, 3, 4, 5, 2 c. 1, 4, 3, 5, 2 d. 1, 4, 3, 2, 5 MULTIPLE RESPONSE 1. A nurse is developing nursing diagnoses for a group of patients. Which nursing diagnoses will the nurse use? (Select all that apply.) a. Anxiety related to barium enema b. Impaired gas exchange related to asthma c. Impaired physical mobility related to incisional pain d. Nausea related to adverse effect of cancer medication e. Risk for falls related to nursing assistive personnel leaving bedrail down , D Chapter 18: Planning Nursing Care 1. The nurse completes a thorough assessment of a patient and analyzes the data to identify nursing diagnoses. Which step will the nurse take next in the nursing process? a. Assessment b. Diagnosis c. Planning d. Implementation 2. A patient’s plan of care includes the goal of increasing mobility this shift. As the patient is ambulating to the bathroom at the beginning of the shift, the patient suffers a fall. Which initialaction will the nurse take next to revise the plan of care? a. Consult physical therapy. b. Establish a new plan of care. c. Set new priorities for the patient. d. Assess the patient. 3. Which information indicates a nurse has a good understanding of a goal? a. It is a statement describing the patient’s accomplishments without a time restriction. b. It is a realistic statement predicting any negative responses to treatments. c. It is a broad statement describing a desired change in a patient’s behavior. d. It is a measurable change in a patient’s physical state. 4. A nurse is developing a care plan for a patient with a pelvic fracture on bed rest. Which goal statement is realistic for the nurse to assign to this patient? a. Patient will increase activity level this shift. b. Patient will turn side to back to side with assistance every 2 hours. c. Patient will use the walker correctly to ambulate to the bathroom as needed. d. Patient will use a sliding board correctly to transfer to the bedside commode as needed. 5. The following statements are on a patient’s nursing care plan. Which statement will the nurse use as an outcome for a goal of care? a. The patient will verbalize a decreased pain level less than 3 on a 0 to 10 scale by the end of this shift. b. The patient will demonstrate increased tolerance to activity over the next month. c. The patient will understand needed dietary changes by discharge. d. The patient will demonstrate increased mobility in 2 days. 6. A charge nurse is reviewing outcome statements using the SMART approach. Which patient outcome statement will the charge nurse praise to the new nurse? a. The patient will ambulate in hallways. b. The nurse will monitor the patient’s heart rhythm continuously this shift. c. The patient will feed self at all mealtimes today without reports of shortness of breath. d. The nurse will administer pain medication every 4 hours to keep the patient free from discomfort. 7. A nursing assessment for a patient with a spinal cord injury leads to several pertinent nursing diagnoses. Which nursing diagnosis is the highest priority for this patient? a. Risk for impaired skin integrity b. Risk for infection c. Spiritual distress d. Reflex urinary incontinence 8. The new nurse is caring for six patients in this shift. After completing their assessments, the nurse asks where to begin in developing care plans for these patients. Which statement is an appropriate suggestion by another nurse? a. “Choose all the interventions and perform them in order of time needed for each one.” b. “Make sure you identify the scientific rationale for each intervention first.” c. “Decide on goals and outcomes you have chosen for the patients.” d. “Begin with the highest priority diagnoses, then select appropriate interventions.” 9. A patient’s son decides to stay at the bedside while his father is confused. When developing the plan of care for this patient, what should the nurse do? a. Individualize the care plan only according to the patient’s needs. b. Request that the son leave at bedtime, so the patient can rest. c. Suggest that a female member of the family stay with the patient. d. Involve the son in the plan of care as much as possible. 10. A nurse is caring for a patient with a nursing diagnosis of Constipation related to slowed gastrointestinal motility secondary to pain medications. Which outcome is most appropriate for the nurse to include in the plan of care? a. Patient will have one soft, formed bowel movement by end of shift. b. Patient will walk unassisted to bathroom by the end of shift. c. Patient will be offered laxatives or stool softeners this shift. d. Patient will not take any pain medications this shift. 11. The nurse performs an intervention for a collaborative problem. Which type of intervention did the nurse perform? a. Dependent b. Independent c. Interdependent d. Physician-initiated 12. A registered nurse administers pain medication to a patient suffering from fractured ribs. Which type of nursing intervention is this nurse implementing? a. Collaborative b. Independent c. Interdependent d. Dependent 13. Which action indicates the nurse is using a PICOT question to improve care for a patient? a. Practices nursing based on the evidence presented in court b. Implements interventions based on scientific research c. Uses standardized care plans for all patients. d. Plans care based on tradition 14. A nurse is developing a care plan. Which intervention is most appropriate for the nursing diagnostic statement Risk for loneliness related to impaired verbal communication? a. Provide the patient with a writing board each shift. b. Obtain an interpreter for the patient as soon as possible. c. Assist the patient in performing swallowing exercises each shift. d. Ask the family to provide a sitter to remain with the patient at all times. 15. A nurse is completing a care plan. Which intervention is most appropriate for the nursing diagnostic statement Impaired skin integrity related to shearing forces? a. Administer pain medication every 4 hours as needed. b. Turn the patient every 2 hours, even hours. c. Monitor vital signs, especially rhythm. d. Keep the bed side rails up at all times. 16. A patient has reduced muscle strength following a left-sided stroke and is at risk for falling. Which intervention is most appropriate for the nursing diagnostic statement Risk for falls? a. Keep all side rails down at all times. b. Encourage patient to remain in bed most of the shift. c. Place patient in room away from the nurses’ station if possible. d. Assist patient into and out of bed every 4 hours or as tolerated. 17. Which action will the nurse take after the plan of care for a patient is developed? a. Place the original copy in the chart, so it cannot be tampered with or revised. b. Communicate the plan to all health care professionals involved in the patient’s care. c. File the plan of care in the administration office for legal examination. d. Send the plan of care to quality assurance for review. 18. A nurse is preparing to make a consult. In which order, beginning with the first step, will the nurse take? 1. Identify the problem. 2. Discuss the findings and recommendation. 3. Provide the consultant with relevant information about the problem. 4. Contact the right professional, with the appropriate knowledge and expertise. 5. Avoid bias by not providing a lot of information based on opinion to the consultant. a. 1, 4, 3, 5, 2 b. 4, 1, 3, 2, 5 c. 1, 4, 5, 3, 2 d. 4, 3, 1, 5, 2 19. A hospital’s wound nurse consultant made a recommendation for nurses on the unit about how to care for the patient’s dressing changes. Which action should the nurses take next? a. Include dressing change instructions and frequency in the care plan. b. Assume that the wound nurse will perform all dressing changes. c. Request that the health care provider look at the wound. d. Encourage the patient to perform the dressing changes. MULTIPLE RESPONSE 1. A nurse is planning care for a patient with a nursing diagnosis of Impaired skin integrity. The patient needs many nursing interventions, including a dressing change, several intravenous antibiotics, and a walk. Which factors does the nurse consider when prioritizing interventions? (Select all that apply.) a. Rank all the patient’s nursing diagnoses in order of priority. b. Do not change priorities once they’ve been established. c. Set priorities based solely on physiological factors. d. Consider time as an influencing factor. e. Utilize critical thinking. , D, E 2. A nurse is teaching the staff about the benefits of Nursing Outcomes Classification. Which information should the nurse include in the teaching session? (Select all that apply.) a. Includes seven domains for level 1 b. Uses an easy 3-point Likert scale c. Adds objectivity to judging a patient’s progress d. Allows choice in which interventions to choose e. Measures nursing care on a national and international level , E Chapter 19: Implementing Nursing Care 1. A nurse is providing nursing care to patients after completing a care plan from nursing diagnoses. In which step of the nursing process is the nurse? a. Assessment b. Planning c. Implementation d. Evaluation 2. The nurse is teaching a new nurse about protocols. Which information from the new nurse indicates a correct understanding of the teaching? a. Protocols are guidelines to follow that replace the nursing care plan. b. Protocols assist the clinician in making decisions and choosing interventions for specific health care problems or conditions. c. Protocols are policies designating each nurse’s duty according to standards of care and a code of ethics. d. Protocols are prescriptive order forms that help individualize the plan of care. 3. The standing orders for a patient include acetaminophen 650 mg every 4 hours prn for headache. After assessing the patient, the nurse identifies the need for headache relief and determines that the patient has not had acetaminophen in the past 4 hours. Which action will the nurse take next? a. Administer the acetaminophen. b. Notify the health care provider to obtain a verbal order. c. Direct the nursing assistive personnel to give the acetaminophen. d. Perform a pain assessment only after administering the acetaminophen. 4. Which action indicates a nurse is using critical thinking for implementation of nursing care to patients? a. Determines whether an intervention is correct and appropriate for the given situation b. Reads over the steps and performs a procedure despite lack of clinical competency c. Establishes goals for a particular patient without assessment d. Evaluates the effectiveness of interventions 5. A nurse is reviewing a patient’s care plan. Which information will the nurse identify as a nursing intervention? a. The patient will ambulate in the hallway twice this shift using crutches correctly. b. Impaired physical mobility related to inability to bear weight on right leg. c. Provide assistance while the patient walks in the hallway twice this shift with crutches. d. The patient is unable to bear weight on right lower extremity. 6. A patient recovering from a leg fracture after a fall reports having dull pain in the affected leg and rates it as a 7 on a 0 to 10 scale. The patient is not able to walk around in the room with crutches because of leg discomfort. Which nursing intervention is priority? a. Assist the patient to walk in the room with crutches. b. Obtain a walker for the patient. c. Consult physical therapy. d. Administer pain medication. 7. The nurse is caring for a patient who requires a complex dressing change. While in the patient’s room, the nurse decides to change the dressing. Which action will the nurse take just before changing the dressing? a. Gathers and organizes needed supplies b. Decides on goals and outcomes for the patient c. Assesses the patient’s readiness for the procedure d. Calls for assistance from another nursing staff member 8. A patient visiting with family members in the waiting area tells the nurse “I don’t feel good, especially in the stomach.” What should the nurse do? a. Request that the family leave, so the patient can rest. b. Ask the patient to return to the room, so the nurse can inspect the abdomen. c. Ask the patient when the last bowel movement was and to lie down on the sofa. d. Tell the patient that the dinner tray will be ready in 15 minutes and that may help the stomach feel better. 9. A newly admitted patient who is morbidly obese asks the nurse for assistance to the bathroom for the first time. Which action should the nurse take initially? a. Ask for at least two other assistive personnel to come to the room. b. Medicate the patient to alleviate discomfort while ambulating. c. Review the patient’s activity orders. d. Offer the patient a walker. 10. A new nurse is working in a unit that uses interdisciplinary collaboration. Which action will the nurse take? a. Act as a leader of the health care team. b. Develop good communication skills. c. Work solely with nurses. d. Avoid conflict. 11. Which action should the nurse take first during the initial phase of implementation? a. Determine patient outcomes and goals. b. Prioritize patient’s nursing diagnoses. c. Evaluate interventions. d. Reassess the patient. 12. Vital signs for a patient reveal a high blood pressure of 187/100. Orders state to notify the health care provider for diastolic blood pressure greater than 90. What is the nurse’s firstaction? a. Follow the clinical protocol for a stroke. b. Review the most recent lab results for the patient’s potassium level. c. Assess the patient for other symptoms or problems, and then notify the health care provider. d. Administer an antihypertensive medication from the stock supply, and then notify the health care provider. 13. Which initial intervention is most appropriate for a patient who has a new onset of chest pain? a. Reassess the patient. b. Notify the health care provider. c. Administer a prn medication for pain. d. Call radiology for a portable chest x-ray. 14. A nurse is making initial rounds on patients. Which intervention for a patient with poor wound healing should the nurse perform first? a. Reinforce the wound dressing as needed with 4 × 4 gauze. b. Perform the ordered dressing change twice daily. c. Observe wound appearance and edges. d. Document wound characteristics. 15. The nurse establishes trust and talks with a school-aged patient before administering an injection. Which type of implementation skill is the nurse using? a. Cognitive b. Interpersonal c. Psychomotor d. Judgmental 16. The nurse inserts an intravenous (IV) catheter using the correct technique and following the recommended steps according to standards of care and hospital policy. Which type of implementation skill is the nurse using? a. Cognitive b. Interpersonal c. Psychomotor d. Judgmental 17. A staff development nurse is providing an inservice for other nurses to educate them about the Nursing Interventions Classification (NIC) system. During the inservice, which statement made by one of the nurses in the room requires the staff development nurse to clarify the information provided? a. “This system can help medical students determine the cost of the care they provide to patients.” b. “If the nursing department uses this system, communication among nurses who work throughout the hospital may be enhanced.” c. “We could use this system to help organize orientation for new nursing employees because we can better explain the nursing interventions we use most frequently on our unit.” d. “The NIC system provides one way to improve safe and effective documentation in the hospital’s electronic health record.” 18. The nurse is intervening for a family member with role strain. Which direct care nursing intervention is most appropriate? a. Assisting with activities of daily living b. Counseling about respite care options c. Teaching range-of-motion exercises d. Consulting with a social worker 19. The nurse is intervening for a patient that has a risk for a urinary infection. Which direct care nursing intervention is most appropriate? a. Teaches proper handwashing technique b. Properly cleans the patient’s toilet c. Transports urine specimen to the lab d. Informs the oncoming nurse during hand-off 20. The nurse is revising the care plan. In which order will the nurse perform the tasks, beginning with the first step? 1. Revise specific interventions. 2. Revise the assessment column. 3. Choose the evaluation method. 4. Delete irrelevant nursing diagnoses. a. 2, 4, 1, 3 b. 4, 2, 1, 3 c. 3, 4, 2, 1 d. 4, 2, 3, 1 Modification of an existing written care plan includes four steps: 1. Revise data in the assessment column to reflect the patient’s current status. Date any new data to inform other members of the health care team of the time that the change occurred. 2. Revise the nursing diagnoses. Delete nursing diagnoses that are no longer relevant and add and date any new diagnoses. Revise related factors and the patient’s goals, outcomes, and priorities. Date any revisions. 3. Revise specific interventions that correspond to the new nursing diagnoses and goals. Be sure that revisions reflect the patient’s present status. 4. Choose the method of evaluation for determining whether you achieved patient outcomes. MULTIPLE RESPONSE 1. A nurse is implementing interventions for a group of patients. Which actions are nursing interventions? (Select all that apply.) a. Order chest x-ray for suspected arm fracture. b. Prescribe antibiotics for a wound infection. c. Reposition a patient who is on bed rest. d. Teach a patient preoperative exercises. e. Transfer a patient to another hospital unit. , D, E 2. A nurse is providing nursing care to a group of patients. Which actions are direct care interventions? (Select all that apply.) a. Ambulating a patient b. Inserting a feeding tube c. Performing resuscitation d. Documenting wound care e. Teaching about medications , B, C, E 3. A nurse is preparing to carry out interventions. Which resources will the nurse make sure are available? (Select all that apply.) a. Equipment b. Safe environment c. Confidence d. Assistive personnel e. Creativity , B, D 4. Which interventions are appropriate for a patient with diabetes and poor wound healing? (Select all that apply.) a. Perform dressing changes twice a day as ordered. b. Teach the patient about signs and symptoms of infection. c. Instruct the family about how to perform dressing changes. d. Gently refocus patient from discussing body image changes. e. Administer medications to control the patient’s blood sugar as ordered. , B, C, E Chapter 20: Evaluation 1. A nurse determines that the patient’s condition has improved and has met expected outcomes. Which step of the nursing process is the nurse exhibiting? a. Assessment b. Planning c. Implementation d. Evaluation 2. A nurse completes a thorough database and carries out nursing interventions based on priority diagnoses. Which action will the nurse take next? a. Assessment b. Planning c. Implementation d. Evaluation 3. A new nurse asks the preceptor to describe the primary purpose of evaluation. Which statement made by the nursing preceptor is most accurate? a. “An evaluation helps you determine whether all nursing interventions were completed.” b. “During evaluation, you determine when to downsize staffing on nursing units.” c. “Nurses use evaluation to determine the effectiveness of nursing care.” d. “Evaluation eliminates unnecessary paperwork and care planning.” 4. After assessing the patient and identifying the need for headache relief, the nurse administers acetaminophen for the patient’s headache. Which action by the nurse is priorityfor this patient? a. Eliminate headache from the nursing care plan. b. Direct the nursing assistive personnel to ask if the headache is relieved. c. Reassess the patient’s pain level in 30 minutes. d. Revise the plan of care. 5. A nurse is getting ready to discharge a patient who has a problem with physical mobility. What does the nurse need to do before discontinuing the patient’s plan of care? a. Determine whether the patient has transportation to get home. b. Evaluate whether patient goals and outcomes have been met. c. Establish whether the patient has a follow-up appointment scheduled. d. Ensure that the patient’s prescriptions have been filled to take home. 6. The nurse is evaluating whether patient goals and outcomes have been met for a patient with physical mobility problems due to a fractured leg. Which finding indicates the patient has met an expected outcome? a. The nurse provides assistance while the patient is walking in the hallways. b. The patient is able to ambulate in the hallway with crutches. c. The patient will deny pain while walking in the hallway. d. The patient’s level of mobility will improve. 7. The nurse is evaluating whether a patient’s turning schedule was effective in preventing the formation of pressure ulcers. Which finding indicates success of the turning schedule? a. Staff documentation of turning the patient every 2 hours b. Presence of redness only on the heels of the patient c. Patient’s eating 100% of all meals d. Absence of skin breakdown 8. A nurse has instituted a turn schedule for a patient to prevent skin breakdown. Upon evaluation, the nurse finds that the patient has a stage II pressure ulcer on the buttocks. Which action will the nurse take next? a. Reassess the patient and situation. b. Revise the turning schedule to increase the frequency. c. Delegate turning to the nursing assistive personnel. d. Apply medication to the area of skin that is broken down. 9. A new nurse is confused about using evaluative measures when caring for patients and asks the charge nurse for an explanation. Which response by the charge nurse is most accurate? a. “Evaluative measures are multiple-page documents used to evaluate nurse performance.” b. “Evaluative measures include assessment data used to determine whether patients have met their expected outcomes and goals.” c. “Evaluative measures are used by quality assurance nurses to determine the progress a nurse is making from novice to expert nurse.” d. “Evaluative measures are objective views for completion of nursing interventions.” 10. The nurse is caring for a patient who has an open wound and is evaluating the progress of wound healing. Which priority action will the nurse take? a. Ask the nursing assistive personnel if the wound looks better. b. Document the progress of wound healing as “better” in the chart. c. Measure the wound and observe for redness, swelling, or drainage. d. Leave the dressing off the wound for easier access and more frequent assessments. 11. The nurse is caring for a patient who has an order to change a dressing twice a day, at 0600 and 1800. At 1400, the nurse notices that the dressing is saturated and leaking. What is the nurse’s next action? a. Wait and change the dressing at 1800 as ordered. b. Revise the plan of care and change the dressing now. c. Reassess the dressing and the wound in 2 hours. d. Discontinue the plan of care for wound care. 12. A goal for a patient with diabetes is to demonstrate effective coping skills. Which patient behavior will indicate to the nurse achievement of this outcome? a. States feels better after talking with family and friends b. Consumes high-carbohydrate foods when stressed c. Dislikes the support group meetings d. Spends most of the day in bed 13. A nurse is providing education to a patient about self-administering subcutaneous injections. The patient demonstrates the self-injection. Which type of indicator did the nurse evaluate? a. Health status b. Health behavior c. Psychological self-control d. Health service utilization 14. A nurse is evaluating the goal of acceptance of body image in a young teenage girl. Which statement made by the patient is the best indicator of progress toward the goal? a. “I’m worried about what those other girls will think of me.” b. “I can’t wear that color. It makes my hips stick out.” c. “I’ll wear the blue dress. It matches my eyes.” d. “I will go to the pool next summer.” 15. A nurse is evaluating goals and expected outcomes for a confused patient. Which finding indicates positive progress toward resolving the confusion? a. Patient wanders halls at night. b. Patient’s side rails are up with bed alarm activated. c. Patient denies pain while ambulating with assistance. d. Patient correctly states names of family members in the room. 16. A nurse identifies a fall risk when assessing a patient upon admission. The nurse and the patient agree that the goal is for the patient to remain free from falls. However, the patient fell just before shift change. Which action is the nurse’s priority when evaluating the patient? a. Identify factors interfering with goal achievement. b. Counsel the nursing assistive personnel on duty when the patient fell. c. Remove the fall risk sign from the patient’s door because the patient has suffered a fall. d. Request that the more experienced charge nurse complete the documentation about the fall. 17. A patient was recently diagnosed with pneumonia. The nurse and the patient have established a goal that the patient will not experience shortness of breath with activity in 3 days with an expected outcome of having no secretions present in the lungs in 48 hours. Which evaluative measure will the nurse use to demonstrate progress toward this goal? a. No sputum or cough present in 4 days b. Congestion throughout all lung fields in 2 days c. Shallow, fast respirations 30 breaths per minute in 1 day d. Lungs clear to auscultation following use of inhaler 18. A nurse is evaluating an expected outcome for a patient that states heart rate will be less than 80 beats/min by 12/3. Which finding will alert the nurse that the goal has been met? a. Heart rate 78 beats/min on 12/3 b. Heart rate 78 beats/min on 12/4 c. Heart rate 80 beats/min on 12/3 d. Heart rate 80 beats/min on 12/4 19. A nurse is modifying a patient’s care plan after evaluation of patient care. In which order, starting with the first step, will the nurse perform the tasks? 1. Revise nursing diagnosis. 2. Reassess blood pressure reading. 3. Retake blood pressure after medication. 4. Administer new blood pressure medication. 5. Change goal to blood pressure less than 140/90. a. 1, 5, 2, 4, 3 b. 2, 1, 5, 4, 3 c. 4, 3, 1, 5, 2 d. 5, 4, 5, 1, 2 MULTIPLE RESPONSE 1. A nurse is caring for a group of patients. Which evaluative measures will the nurse use to determine a patient’s responses to nursing care? (Select all that apply.) a. Observations of wound healing b. Daily blood pressure measurements c. Findings of respiratory rate and depth d. Completion of nursing interventions e. Patient’s subjective report of feelings about a new diagnosis of cancer , B, C, E 2. Which nursing actions will the nurse perform in the evaluation phase of the nursing process? (Select all that apply.) a. Set priorities for patient care. b. Determine whether outcomes or standards are met. c. Ambulate patient 25 feet in the hallway. d. Document results of goal achievement. e. Use self-reflection and correct errors. , D, E Chapter 21: Managing Patient Care 1. A registered nurse (RN) is the group leader of practical nurses and nursing assistive personnel. Which nursing care model is the RN using? a. Case management b. Total patient care c. Primary nursing d. Team nursing 2. A nurse is overseeing the care of patients with severe diabetes and patients with heart failure to improve cost-effectiveness and quality of care. Which nursing care delivery model is the nurse using? a. Team nursing b. Total patient care c. Primary nursing d. Case management 3. A nurse is working in an intensive care unit (critical care). Which type of nursing care delivery model will this nurse most likely use? a. Team nursing b. Total patient care c. Primary nursing d. Case-management 4. A nurse manager discovers that the readmission rate of hospitalized patients is very high on the hospital unit. The nurse manager desires improved coordination of care and accountability for cost-effective quality care. Which nursing care delivery model is best suited for these needs? a. Team nursing b. Total patient care c. Primary nursing d. Case management 5. A nurse is working in a facility that has fewer directors with managers and staff able to make shared decisions. In which type of organizational structure is the nurse employed? a. Delegation b. Research-based c. Decentralization d. Philosophy of care 6. A staff member verbalizes satisfaction in working on a particular nursing unit because of the freedom of choice and responsibility for the choices. This nurse highly values which element of shared decision making? a. Authority b. Autonomy c. Responsibility d. Accountability 7. A staff nurse delegates a task to a nursing assistive personnel (NAP), knowing that the NAP has never performed the task before. As a result, the patient is injured, and the nurse defensively states that the NAP should have known how to perform such a simple task. Which element of the decision-making process is the nurse lacking? a. Authority b. Autonomy c. Responsibility d. Accountability 8. A nurse manager sent one of the staff nurses on the unit to a conference about new, evidence-based wound care techniques. The nurse manager asks the staff nurse to prepare a poster to present at the next unit meeting, which will be mandatory for all nursing staff on the unit. Which type of opportunity is the nurse manager providing for the staff? a. Staff education b. Interprofessional collaboration c. Providing a professional shared governance council d. Establishing a nursing practice committee 9. A nurse is making a home visit and discovers that a patient’s wound infection has gotten worse. The nurse cleans and redresses the wound. What should the nurse do next? a. Notify the health care provider of the findings before leaving the home. b. Ask the home health facility nurse manager to contact the health care provider. c. Document the findings and confirm with the patient the date of the next home visit. d. Tell the patient that the health care provider will be notified before the next home visit. 10. A nurse manager conducts rounds on the unit and discovers that expired stock medicine is still in the cabinet despite the e-mail that was sent stating that it had to be discarded. The staff nurse dress code is not being adhered to as requested in the same e-mail. Several staff nurses deny having received the e-mail. Which action should the nurse manager take? a. Close the staff lounge. b. Enforce a stricter dress code. c. Include the findings on each staff member’s annual evaluation. d. Place a hard copy of announcements and unit policies in each staff member’s mailbox. 11. A new nurse expresses frustration at not being to complete all interventions for a group of patients in a timely manner. The nurse leaves the rounds report sheets at the nurse’s station when caring for patients and reports having to go back and forth between rooms for equipment and supplies. Which type of skill does the nurse need? a. Interpersonal communication b. Clinical decision making c. Organizational d. Evaluation 12. Which approach will be most appropriate for a nurse to take when faced with the challenge of performing many tasks in one shift? a. Do as much as possible by oneself before seeking assistance from others. b. Evaluate the effectiveness of all tasks when all tasks are completed. c. Complete one task before starting another task. d. Delegate tasks the nurse does not like doing. 13. Which assessment of a patient who is 1 day postsurgery to repair a hip fracture requires immediate nursing intervention? a. Patient ate 40% of clear liquid breakfast. b. Patient’s oral temperature is 98.9° F. c. Patient states, “I did not realize I would be so tired after this surgery.” d. Patient reports severe pain 30 minutes after receiving pain medication. 14. A nurse has a transactional leader as a manager. Which finding will the nurse anticipate from working with this leader? a. Increased turnover rate b. Increased patient mortality rate c. Increased rate of medication errors d. Increased level of patient satisfaction 15. A patient with an indwelling urinary catheter has been given a bed bath by a new nursing assistive personnel. The nurse evaluating the cleanliness of the patient notices crusting at the urinary meatus. Which action should the nurse take next? a. Ask the nursing assistive personnel to observe while the nurse performs catheter care. b. Leave the room and ask the nursing assistive personnel to go back and perform proper catheter care. c. Tell the nursing assistive personnel that catheter care is sloppy. d. Remove the catheter. 16. A nurse is prioritizing care for four patients. Which patient should the nurse see first? a. A patient needing teaching about medications b. A patient with a healed abdominal incision c. A patient with a slight temperature d. A patient with difficulty breathing 17. A nursing assistive personnel (NAP) reports seeing a reddened area on the patient’s hip while bathing the patient. Which action should the nurse take? a. Request a wound nurse consult. b. Go to the patient’s room to assess the patient’s skin. c. Document the finding per the NAP’s report. d. Ask the NAP to apply a dressing over the reddened area. 18. A nurse is assigned to care for the following patients who all need vital signs taken right now. Which patient is most appropriate for the nurse to delegate vital sign measurement to the nursing assistive personnel (NAP)? a. Patient scheduled for a procedure in the nuclear medicine department b. Patient transferring from the intensive care unit (ICU) c. Patient returning from a cardiac catheterization d. Patient returning from hip replacement surgery 19. Which staff member does the nurse assign to provide morning care for an older-adult patient who requires assistance with activities of daily living? a. Licensed practical nurse b. Cardiac monitor technician c. Nursing assistive personnel (NAP) d. Another registered nurse on the floor MULTIPLE RESPONSE 1. A nurse uses the five rights of delegation when providing care. Which “rights” did the nurse use? (Select all that apply.) a. Right task b. Right person c. Right direction d. Right supervision e. Right circumstances f. Right cost-effectiveness , B, C, D, E MATCHING A nurse is prioritizing care. Match the level of priority to the patients. a. Patient that needs to be turned to prevent pneumonia b. Patient with acute asthma attack c. Patient who will be discharged in 2 days who needs teaching 1. High priority 2. Intermediate priority 3. Low priority 1.ANS:BDIF:Apply (application)REF:284 OBJ: Discuss ways to apply clinical care coordination skills in nursing practice. TOPlanningMSC:Management of Care 2.ANS:ADIF:Apply (application)REF:284 OBJ: Discuss ways to apply clinical care coordination skills in nursing practice. TOPlanningMSC:Management of Care 3.ANS:CDIF:Apply (application)REF:284 OBJ: Discuss ways to apply clinical care coordination skills in nursing practice. TOPlanningMSC:Management of Care Chapter 22: Ethics and Values 1. Four patients in labor all request epidural analgesia to manage their pain at the same time. Which ethical principle is most compromised when only one nurse anesthetist is on call? a. Justice b. Fidelity c. Beneficence d. Nonmaleficence 2. The patient reports to the nurse of being afraid to speak up regarding a desire to end care for fear of upsetting spouse and children. Which principle in the nursing code of ethics ensures that the nurse will promote the patient’s cause? a. Advocacy b. Responsibility c. Confidentiality d. Accountability 3. The patient’s son requests to view documentation in the medical record. What is the nurse’sbest response to this request? a. “I’ll be happy to get that for you.” b. “You are not allowed to look at it.” c. “You will need your mother’s permission.” d. “I cannot let you see the chart without a doctor’s order.” 4. When professionals work together to solve ethical dilemmas, nurses must examine their own values. What is the best rationale for this step? a. So fact is separated from opinion b. So different perspectives are respected c. So judgmental attitudes can be provoked d. So the group identifies the one correct solution 5. A nurse is experiencing an ethical dilemma with a patient. Which information indicates the nurse has a correct understanding of the primary cause of ethical dilemmas? a. Unequal power b. Presence of conflicting values c. Judgmental perceptions of patients d. Poor communication with the patient 6. The nurse questions a health care provider’s decision to not tell the patient about a cancer diagnosis. Which ethical principle is the nurse trying to uphold for the patient? a. Consequentialism b. Autonomy c. Fidelity d. Justice 7. The nurse finds it difficult to care for a patient whose advance directive states that no extraordinary resuscitation measures should be taken. Which step may help the nurse to find resolution in this assignment? a. Scrutinize personal values. b. Call for an ethical committee consult. c. Decline the assignment on religious grounds. d. Convince the family to challenge the directive. 8. The nurse values autonomy above all other principles. Which patient assignment will the nurse find most difficult to accept? a. Older-adult patient who requires dialysis b. Teenager in labor who requests epidural anesthesia c. Middle-aged father of three with an advance directive declining life support d. Family elder who is making the decisions for a young-adult female member 9. A nurse must make an ethical decision concerning vulnerable patient populations. Which philosophy of health care ethics would be particularly useful for this nurse? a. Teleology b. Deontology c. Utilitarianism d. Feminist ethics 10. A nurse agrees with regulations for mandatory immunizations of children. The nurse believes that immunizations prevent diseases as well as prevent spread of the disease to others. Which ethical framework is the nurse using? a. Deontology b. Ethics of care c. Utilitarianism d. Feminist ethics 11. The nurse has become aware of missing narcotics in the patient care area. Which ethical principle obligates the nurse to report the missing medications? a. Advocacy b. Responsibility c. Confidentiality d. Accountability 12. A young woman who is pregnant with a fetus exposed to multiple teratogens consents to have her fetus undergo serial PUBS (percutaneous umbilical blood sampling) to examine how exposure affects the fetus over time. Although these tests will not improve the fetus’s outcomes and will expose it to some risks, the information gathered may help infants in the future. Which ethical principle is at greatest risk? a. Fidelity b. Autonomy c. Beneficence d. Nonmaleficence 13. A nurse is discussing quality of life issues with another colleague. Which topic will the nurse acknowledge for increased attention paid to quality of life concerns? a. Health care disparities b. Aging of the population c. Abilities of disabled persons d. Health care financial reform 14. Which action by the nurse indicates a safe and efficient use of social networks? a. Promotes support for a local health charity b. Posts a picture of a patient’s infected foot c. Vents about a patient problem at work d. Friends a patient 15. The nurse is caring for a dying patient. Which intervention is considered futile? a. Giving pain medication for pain b. Providing oral care every 5 hours c. Administering the influenza vaccine d. Supporting lower extremities with pillows 16. During a severe respiratory epidemic, the local health care organizations decide to give health care workers priority access to ventilators over other members of the community who also need that resource. Which philosophy would give the strongest support for this decision? a. Deontology b. Utilitarianism c. Ethics of care d. Feminist ethics 17. A nurse is teaching a patient and family about quality of life. Which information should the nurse include in the teaching session about quality of life? a. It is deeply social. b. It is hard to define. c. It is an observed measurement for most people. d. It is consistent and stable over the course of one’s lifetime. 18. The nurse is caring for a patient supported with a ventilator who has been unresponsive since arrival via ambulance 8 days ago. The patient has not been identified, and no family members have been found. The nurse is concerned about the plan of care regarding maintenance or withdrawal of life support measures. Place the steps the nurse will use to resolve this ethical dilemma in the correct order. 1. The nurse identifies possible solutions or actions to resolve the dilemma. 2. The nurse reviews the medical record, including entries by all health care disciplines, to gather information relevant to this patient’s situation. 3. Health care providers use negotiation to redefine the patient’s plan of care. 4. The nurse evaluates the plan and revises it with input from other health care providers as necessary. 5. The nurse examines the issue to clarify opinions, values, and facts. 6. The nurse states the problem. a. 6, 1, 2, 5, 4, 3 b. 5, 6, 2, 3, 4, 1 c. 1, 2, 5, 4, 3, 6 d. 2, 5, 6, 1, 3, 4 Step 1. Gather as much information as possible that is relevant to the case. Step 2. Examine and determine your values about the issues. Step 3. Verbalize the problem. Step 4. Consider possible courses of action. Step 5. Negotiate the outcome. Step 6. Evaluate the action. MULTIPLE RESPONSE 1. A nurse is a member of the ethics committee. Which purposes will the nurse fulfill in this committee? (Select all that apply.) a. Education b. Case consultation c. Purchasing power d. Direct patient care e. Policy recommendation , B, E Chapter 23: Legal Implications in Nursing Practice 1. A newly hired experienced nurse is preparing to change a patient’s abdominal dressing and hasn’t done it before at this hospital. Which action by the nurse is best? a. Have another nurse do it so the correct method can be viewed. b. Change the dressing using the method taught in nursing school. c. Ask the patient how the dressing change has been recently done. d. Check the policy and procedure manual for the facility’s method. 2. A new nurse notes that the health care unit keeps a listing of patient names in a closed book behind the front desk of the nursing station so patients can be located easily. Which action ismost appropriate for the nurse to take? a. Talk with the nurse manager about the listing being a violation of the Health Insurance Portability and Accountability Act (HIPAA). b. Use the book as needed while keeping it away from individuals not involved in patient care. c. Move the book to the upper ledge of the nursing station for easier access. d. Ask the nurse manager to move the book to a more secluded area. 3. A 17-year-old patient, dying of heart failure, wants to have organs removed for transplantation after death. Which action by the nurse is correct? a. Instruct the patient to talk with parents about the desire to donate organs. b. Notify the health care provider about the patient’s desire to donate organs. c. Prepare the organ donation form for the patient to sign while still oriented. d. Contact the United Network for Organ Sharing after talking with the patient. 4. An obstetric nurse comes across an automobile accident. The driver seems to have a crushed upper airway, and while waiting for emergency medical services to arrive, the nurse makes a cut in the trachea and inserts a straw from a purse to provide an airway. The patient survives and has a permanent problem with vocal cords, making it difficult to talk. Which statement is true regarding the nurse’s performance? a. The nurse acted appropriately and saved the patient’s life. b. The nurse stayed within the guidelines of the Good Samaritan Law. c. The nurse took actions beyond those that are standard and appropriate. d. The nurse should have just stayed with the patient and waited for help. 5. A nurse performs cardiopulmonary resuscitation (CPR) on a 92-year-old with brittle bones and breaks a rib during the procedure, which then punctures a lung. The patient recovers completely without any residual problems and sues the nurse for pain and suffering and for malpractice. Which key point will the prosecution attempt to prove against the nurse? a. The CPR procedure was done incorrectly. b. The patient would have died if nothing was done. c. The patient was resuscitated according to the policy. d. The older patient with brittle bones might sustain fractures when chest compressions are done. 6. A recent immigrant who does not speak English is alert and requires hospitalization. What is the initial action that the nurse must take to enable informed consent to be obtained? a. Ask a family member to translate what the nurse is saying. b. Request an official interpreter to explain the terms of consent. c. Notify the nursing manager that the patient doesn’t speak English. d. Use hand gestures and medical equipment while explaining in English. 7. A pediatric oncology nurse floats to an orthopedic trauma unit. Which action should the nurse manager of the orthopedic unit take to enable safe care to be given by this nurse? a. Provide a complete orientation to the functioning of the entire unit. b. Determine patient acuity and care the nurse can safely provide. c. Allow the nurse to choose which mealtime works best. d. Assign nursing assistive personnel to assist with care. 8. While recovering from a severe illness, a hospitalized patient wants to change a living will, which was signed 9 months ago. Which response by the nurse is most appropriate? a. “Check with your admitting health care provider whether a copy is on your chart.” b. “Let me check with someone here in the hospital who can assist you.” c. “You are not allowed to ever change a living will after signing it.” d. “Your living will can be changed only once each calendar year.” 9. A home health nurse notices that a patient’s preschool children are often playing on the sidewalk and in the street unsupervised and repeatedly takes them back to the home and talks with the patient, but the situation continues. Which immediate action by the nurse is mandated by law? a. Contact the appropriate community child protection facility. b. Tell the parents that the authorities will be contacted shortly. c. Take pictures of the children to support the overt child abuse. d. Discuss with both parents about the safety needs of their children. 10. A confused patient with a urinary catheter, nasogastric tube, and intravenous line keeps touching these needed items for care. The nurse has tried to explain to the patient that these lines should not be touched, but the patient continues. Which is the best action by the nurse at this time? a. Apply restraints loosely on the patient’s dominant wrist. b. Notify the health care provider that restraints are needed immediately. c. Try other approaches to prevent the patient from touching these care items. d. Allow the patient to pull out lines to prove that the patient needs to be restrained. 11. A patient with sepsis as a result of long-term leukemia dies 25 hours after admission to the hospital. A full code was conducted without success. The patient had a urinary catheter, an intravenous line, an oxygen cannula, and a nasogastric tube. Which question is the priority for the nurse to ask the family before beginning postmortem care? a. “Is an autopsy going to be done?” b. “Which funeral home do you want to use?” c. “Would you like to assist in bathing your loved one?” d. “Do you want me to remove the lines and tubes before you see your loved one?” 12. Conjoined twins are in the neonatal department of the community hospital until transfer to the closest medical center. A photographer from the local newspaper gets off the elevator on the neonatal floor and wants to take pictures of the infants. Which initial action should the nurse take? a. Escort the cameraman to the neonatal unit while a few pictures are taken quietly. b. Tell the cameraman where the hospital’s public relations department is located. c. Have the cameraman wait for permission from the health care provider. d. Ask the cameraman how the pictures are to be used in the newspaper. 13. A nursing student has been written up several times for being late with providing patient care and for omitting aspects of patient care and not knowing basic procedures that were taught in the skills course one term earlier. The nursing student says, “I don’t understand what the big deal is. As my instructor, you are there to protect me and make sure I don’t make mistakes.” What is the best response from the nursing instructor? a. “You are practicing under the license of the hospital’s insurance.” b. “You are expected to perform at the level of a professional nurse.” c. “You are expected to perform at the level of a prudent nursing student.” d. “You are practicing under the license of the nurse assigned to the patient.” 14. A nurse works full time on the oncology unit at the hospital and works part time on weekends giving immunizations at the local pharmacy. While giving an injection on a weekend, the nurse caused injury to the patient’s arm and is now being sued. How will the hospital’s malpractice insurance provide coverage for this nurse? a. It will provide coverage as long as the nurse followed all procedures, protocols, and policies correctly. b. The hospital’s malpractice insurance covers this nurse only during the time the nurse is working at the hospital. c. As long as the nurse has never been sued before this incident, the hospital’s malpractice insurance will cover the nurse. d. The hospital’s malpractice insurance will provide approximately 50% of the coverage the nurse will need. 15. A female nursing student in the final term of nursing school is overheard by a nursing faculty member telling another student that she got to insert a nasogastric tube in the emergency department while working as a nursing assistant. Which advice is best for the nursing faculty member to give to the nursing student? a. “Just be careful when you are doing new procedures and make sure you are following directions by the nurse.” b. “Review your procedures before you go to work, so you will be prepared to do them if you have a chance.” c. “The nurse should not have allowed you to insert the nasogastric tube because something bad could have happened.” d. “You are not allowed to perform any procedures other than those in your job description even with the nurse’s permission.” MULTIPLE RESPONSE 1. The nurse calculates the medication dose for an infant on the pediatric unit and determines that the dose is twice what it should be based upon the drug book’s information. The pediatrician is contacted and says to administer the medication as ordered. Which actions should the nurse take next? (Select all that apply.) a. Notify the nursing supervisor. b. Administer the medication as ordered. c. Give the amount listed in the drug book. d. Ask the mother to give the drug to her child. e. Check the chain of command policy for such situations. , E 2. The nurse hears a health care provider say to the charge nurse that a certain nurse cannot care for patients because the nurse is stupid and won’t follow orders. The health care provider also writes in the patient’s medical records that the same nurse, by name, is not to care for any of the patients because of incompetence. Which torts has the health care provider committed? (Select all that apply.) a. Libel b. Slander c. Assault d. Battery e. Invasion of privacy , B 3. A patient has approximately 6 months to live and asks about a do not resuscitate (DNR) order. Which statements by the nurse give the patient correct information? (Select all that apply.) a. “You will be resuscitated unless there is a DNR order in the chart.” b. “If you want certain procedures or actions taken or not taken, and you might not be able to tell anyone at the time, you need to complete documents ahead of time that give your health care provider this information.” c. “You will be resuscitated at any time to allow you the longest length of survival.” d. “If you decide you want a DNR order, you will need to talk to your health care provider.” e. “If you travel to another state, your living will should cover your wishes.” , B, D 4. A nurse is teaching the staff about professional negligence or malpractice. Which criteria to establish negligence will the nurse include in the teaching session? (Select all that apply.) a. Injury did not occur. b. That duty was breached. c. Nurse carried out the duty. d. Duty of care was owed to the patient. e. Patient understands benefits and risks of a procedure. , D MATCHING A nurse is discussing nursing actions that can lead to breaches of nursing practice. Match the example to the term it describes. a. Nurse posts about patient’s loud and unruly family members. b. Nurse immediately applies restraints to make patient stay in bed. c. Nurse leaves bed in high position, causing patient to fall and break hip. d. Nurse states that she will wrap a bandage over patient’s mouth if he won’t be quiet. e. Nurse applies abdominal bandage after refusal. f. Nurse gets angry at patient and nurse leaves the hospital. 1. Assault 2. Battery 3. Abandonment 4. False imprisonment 5. Invasion of privacy 6. Malpractice 1.ANS DIF:Apply (application)REF:308-309 OBJ: Analyze nursing actions most often associated in a breach of nursing practice. TOP:ImplementationMSC:Management of Care 2.ANS:EDIF:Apply (application)REF:308-309 OBJ: Analyze nursing actions most often associated in a breach of nursing practice. TOP:ImplementationMSC:Management of Care 3.ANS:FDIF:Apply (application)REF:308-309 OBJ: Analyze nursing actions most often associated in a breach of nursing practice. TOP:ImplementationMSC:Management of Care 4.ANS:BDIF:Apply (application)REF:308-309 OBJ: Analyze nursing actions most often associated in a breach of nursing practice. TOP:ImplementationMSC:Management of Care 5.ANS:ADIF:Apply (application)REF:308-309 OBJ: Analyze nursing actions most often associated in a breach of nursing practice. TOP:ImplementationMSC:Management of Care 6.ANS:CDIF:Apply (application)REF:308-309 OBJ: Analyze nursing actions most often associated in a breach of nursing practice. TOP:ImplementationMSC:Management of Care Chapter 24: Communication 1. Which types of nurses make the best communicators with patients? a. Those who learn effective psychomotor skills b. Those who develop critical thinking skills c. Those who like different kinds of people d. Those who maintain perceptual biases 2. A nurse believes that the nurse-patient relationship is a partnership and that both are equal participants. Which term should the nurse use to describe this belief? a. Critical thinking b. Authentic c. Mutuality d. Attend 3. A nurse wants to present information about flu immunizations to the older adults in the community. Which type of communication should the nurse use? a. Public b. Small group c. Interpersonal d. Intrapersonal 4. A nurse is using therapeutic communication with a patient. Which technique will the nurse use to ensure effective communication? a. Interpersonal communication to change negative self-talk to positive self-talk b. Small group communication to present information to an audience c. Electronic communication to assess a patient in another city d. Intrapersonal communication to build strong teams 5. A nurse is standing beside the patient’s bed. Nurse: How are you doing? Patient: I don’t feel good. Which element will the nurse identify as feedback? a. Nurse b. Patient c. How are you doing? d. I don’t feel good. 6. A nurse is sitting at the patient’s bedside taking a nursing history. Which zone of personal space is the nurse using? a. Socio-consultative b. Personal c. Intimate d. Public 7. A smiling patient angrily states, “I will not cough and deep breathe.” How will the nurse interpret this finding? a. The patient’s denotative meaning is wrong. b. The patient’s personal space was violated. c. The patient’s affect is inappropriate. d. The patient’s vocabulary is poor. 8. The nurse asks a patient where the pain is, and the patient responds by pointing to the area of pain. Which form of communication did the patient use? a. Verbal b. Nonverbal c. Intonation d. Vocabulary 9. A patient has been admitted to the hospital numerous times. The nurse asks the patient to share a personal story about the care that has been received. Which interaction is the nurse using? a. Nonjudgmental b. Socializing c. Narrative d. SBAR 10. Before meeting the patient, a nurse talks to other caregivers about the patient. Which phase of the helping relationship is the nurse in with this patient? a. Preinteraction b. Orientation c. Working d. Termination 11. During the initial home visit, a home health nurse lets the patient know that the visits are expected to end in about a month. Which phase of the helping relationship is the nurse in with this patient? a. Preinteraction b. Orientation c. Working d. Termination 12. A nurse and a patient work on strategies to reduce weight. Which phase of the helping relationship is the nurse in with this patient? a. Preinteraction b. Orientation c. Working d. Termination 13. A nurse uses SBAR when providing a hands-off report to the oncoming shift. What is the rationale for the nurse’s action? a. To promote autonomy b. To use common courtesy c. To establish trustworthiness d. To standardize communication 14. A patient was admitted 2 days ago with pneumonia and a history of angina. The patient is now having chest pain with a pulse rate of 108. Which piece of data will the nurse use for “B” when using SBAR? a. Having chest pain b. Pulse rate of 108 c. History of angina d. Oxygen is needed 15. A patient just received a diagnosis of cancer. Which statement by the nurse demonstrates empathy? a. “Tomorrow will be better.” b. “This must be hard news to hear.” c. “What’s your biggest fear about this diagnosis?” d. “I believe you can overcome this because I’ve seen how strong you are.” 16. A nurse is taking a history on a patient who cannot speak English. Which action will the nurse take? a. Obtain an interpreter. b. Refer to a speech therapist. c. Let a close family member talk. d. Find a mental health nurse specialist. 17. A nurse is using SOLER to facilitate active listening. Which technique should the nurse use for R? a. Relax b. Respect c. Reminisce d. Reassure 18. An older-adult patient is wearing a hearing aid. Which technique should the nurse use to facilitate communication? a. Chew gum. b. Turn off the television. c. Speak clearly and loudly. d. Use at least 14-point print. 19. When making rounds, the nurse finds a patient who is not able to sleep because of surgery in the morning. Which therapeutic response is most appropriate? a. “You will be okay. Your surgeon will talk to you in the morning.” b. “Why can’t you sleep? You have the best surgeon in the hospital.” c. “Don’t worry. The surgeon ordered a sleeping pill to help you sleep.” d. “It must be difficult not to know what the surgeon will find. What can I do to help?” 20. Which situation will cause the nurse to intervene and follow up on the nursing assistive personnel’s (NAP) behavior? a. The nursing assistive personnel is calling the older-adult patient “honey.” b. The nursing assistive personnel is facing the older-adult patient when talking. c. The nursing assistive personnel cleans the older-adult patient’s glasses gently. d. The nursing assistive personnel allows time for the older-adult patient to respond. 21. A confused older-adult patient is wearing thick glasses and a hearing aid. Which intervention is the priority to facilitate communication? a. Focus on tasks to be completed. b. Allow time for the patient to respond. c. Limit conversations with the patient. d. Use gestures and other nonverbal cues. 22. The staff is having a hard time getting an older-adult patient to communicate. Which technique should the nurse suggest the staff use? a. Try changing topics often. b. Allow the patient to reminisce. c. Ask the patient for explanations. d. Involve only the patient in conversations. 23. A nurse is implementing nursing care measures for patients’ special communication needs. Which patient will need the most nursing care measures? a. The patient who is oriented, pain free, and blind b. The patient who is alert, hungry, and has strong self-esteem c. The patient who is cooperative, depressed, and hard of hearing d. The patient who is dyspneic, anxious, and has a tracheostomy 24. A patient is aphasic, and the nurse notices that the patient’s hands shake intermittently. Which nursing action is most appropriate to facilitate communication? a. Use a picture board. b. Use pen and paper. c. Use an interpreter. d. Use a hearing aid. 25. Which behavior indicates the nurse is using a process recording correctly to enhance communication with patients? a. Shows sympathy appropriately b. Uses automatic responses fluently c. Demonstrates passive remarks accurately d. Self-examines personal communication skills 26. A patient says, “You are the worst nurse I have ever had.” Which response by the nurse ismost assertive? a. “I think you’ve had a hard day.” b. “I feel uncomfortable hearing that statement.” c. “I don’t think you should say things like that. It is not right.” d. “I have been checking on you regularly. How can you say that?” MULTIPLE RESPONSE 1. Which behaviors indicate the nurse is using critical thinking standards when communicating with patients? (Select all that apply.) a. Instills faith b. Uses humility c. Portrays self-confidence d. Exhibits supportiveness e. Demonstrates independent attitude , C, E 2. A nurse is implementing nursing care measures for patients with challenging communication issues. Which types of patients will need these nursing care measures? (Select all that apply.) a. A child who is developmentally delayed b. An older-adult patient who is demanding c. A female patient who is outgoing and flirty d. A male patient who is cooperative with treatments e. An older-adult patient who can clearly see small print f. A teenager frightened by the prospect of impending surgery , B, C, F MATCHING A nurse is using AIDET to communicate with patients and families. Match the letters of the acronym to the behavior a nurse will use. a. Nurse describes procedures and tests. b. Nurse lets the patient know how long the procedure will last. c. Nurse recognizes the person with a positive attitude. d. Nurse thanks the patient. e. Nurse tells the patient “I am an RN and will be managing your care.” 1.A 2.I 3.D 4.E 5.T 1.ANS:CDIF:Apply (application)REF:325 OBJ: Demonstrate qualities, behaviors, and communication techniques of professional communication while interacting with patients. TOP: Implementation MSC:Management of Care 2.ANS:EDIF:Apply (application)REF:325 OBJ: Demonstrate qualities, behaviors, and communication techniques of professional communication while interacting with patients. TOP: Implementation MSC:Management of Care 3.ANS:BDIF:Apply (application)REF:325 OBJ: Demonstrate qualities, behaviors, and communication techniques of professional communication while interacting with patients. TOP: Implementation MSC:Management of Care 4.ANS:ADIF:Apply (application)REF:325 OBJ: Demonstrate qualities, behaviors, and communication techniques of professional communication while interacting with patients. TOP: Implementation MSC:Management of Care 5.ANS DIF:Apply (application)REF:325 OBJ: Demonstrate qualities, behaviors, and communication techniques of professional communication while interacting with patients. TOP: Implementation MSC:Management of Care Chapter 29: Infection Prevention and Control 1. The nurse and a new nurse in orientation are caring for a patient with pneumonia. Which statement by the new nurse will indicate a correct understanding of this condition? a. “An infectious disease like pneumonia may not pose a risk to others.” b. “We need to isolate the patient in a private negative-pressure room.” c. “Clinical signs and symptoms are not present in pneumonia.” d. “The patient will not be able to return home.” 2. The patient and the nurse are discussing Rickettsia rickettsii—Rocky Mountain spotted fever. Which patient statement to the nurse indicates understanding regarding the mode of transmission for this disease? a. “When camping, I will use sunscreen.” b. “When camping, I will drink bottled water.” c. “When camping, I will wear insect repellent.” d. “When camping, I will wash my hands with hand gel.” 3. The nurse is providing an educational session for a group of preschool workers. The nurse reminds the group about the most important thing to do to prevent the spread of infection. Which information did the nurse share with the preschool workers? a. Encourage preschool children to eat a nutritious diet. b. Suggest that parents provide a multivitamin to the children. c. Clean the toys every afternoon before putting them away. d. Wash their hands between each interaction with children. 4. The nurse is admitting a patient with an infectious disease process. Which question will bemost appropriate for a nurse to ask about the patient’s susceptibility to this infectious process? a. “Do you have a spouse?” b. “Do you have a chronic disease?” c. “Do you have any children living in the home?” d. “Do you have any religious beliefs that will influence your care?” 5. The patient experienced a surgical procedure, and Betadine was utilized as the surgical prep. Two days postoperatively, the nurse’s assessment indicates that the incision is red and has a small amount of purulent drainage. The patient reports tenderness at the incision site. The patient’s temperature is 100.5° F, and the WBC is 10,500/mm3. Which action should the nurse take first? a. Plan to change the surgical dressing during the shift. b. Utilize SBAR to notify the primary health care provider. c. Reevaluate the temperature and white blood cell count in 4 hours. d. Check to see what solution was used for skin preparation in surgery. 6. The nurse is providing an education session to an adult community group about the effects of smoking on infection. Which information is most important for the nurse to include in the educational session? a. Smoke from tobacco products clings to your clothing and hair. b. Smoking affects the cilia lining the upper airways in the lungs. c. Smoking can affect the color of the patient’s fingernails. d. Smoking tobacco products can be very expensive. 7. A female adult patient presents to the clinic with reports of a white discharge and itching in the vaginal area. A nurse is taking a health history. Which question is the priority? a. “When was the last time you visited your primary health care provider?” b. “Has this condition affected your eating habits in any way?” c. “What medications are you currently taking?” d. “Are you able to sleep at night?” 8. The nurse is caring for a school-aged child who has injured the right leg after a bicycle accident. Which signs and symptoms will the nurse assess for to determine if the child is experiencing a localized inflammatory response? a. Malaise, anorexia, enlarged lymph nodes, and increased white blood cells b. Chest pain, shortness of breath, and nausea and vomiting c. Dizziness and disorientation to time, date, and place d. Edema, redness, tenderness, and loss of function 9. Which interventions utilized by the nurse will indicate the ability to recognize a localized inflammatory response? a. Vigorous range-of-motion exercises b. Turn, cough, and deep breathe c. Orient to date, time, and place d. Rest, ice, and elevation 10. The nurse is caring for a group of medical-surgical patients. Which patient is most at risk for developing an infection? a. A patient who is in observation for chest pain b. A patient who has been admitted with dehydration c. A patient who is recovering from a right total hip surgery d. A patient who has been admitted for stabilization of heart problems 11. The nurse is caring for a patient with leukemia and is preparing to provide fluids through a vascular access (IV) device. Which nursing intervention is a priority in this procedure? a. Review the procedure with the patient. b. Position the patient comfortably. c. Maintain surgical aseptic technique. d. Gather available supplies. 12. The nurse is caring for an adult patient in the clinic who has been evacuated and is a victim of flooding. The nurse teaches the patient about rest, exercise, and eating properly and how to utilize deep breathing and visualization. What is the primary rationale for the nurse’s actions related to the teaching? a. Topics taught are standard information taught during health care visits. b. The patient requested this information to teach the extended family members. c. Stress for long periods of time can lead to exhaustion and decreased resistance to infection. d. These techniques will help the patient manage the pain and loss of personal belongings. 13. The nurse is caring for a patient who is susceptible to infection. Which instruction will the nurse include in an educational session to decrease the risk of infection? a. Teaching the patient about fall prevention b. Teaching the patient to take a temperature c. Teaching the patient to select nutritious foods d. Teaching the patient about the effects of alcohol 14. A diabetic patient presents to the clinic for a dressing change. The wound is located on the right foot and has purulent yellow drainage. Which action will the nurse take to prevent the spread of infection? a. Position the patient comfortably on the stretcher. b. Explain the procedure for dressing change to the patient. c. Review the medication list that the patient brought from home. d. Don gloves and other appropriate personal protective equipment. 15. A patient presents with pneumonia. Which priority intervention should be included in the plan of care for this patient? a. Observe the patient for decreased activity tolerance. b. Assume the patient is in pain and treat accordingly. c. Provide the patient ice chips as requested. d. Maintain the room temperature at 65° F. 16. The nurse is caring for a patient in an intensive care unit who needs a bath. Which priorityaction will the nurse take to decrease the potential for a health care–associated infection? a. Use local anesthetic on reddened areas. b. Use nonallergenic tape on dressings. c. Use a chlorhexidine wash. d. Use filtered water. 17. The infection control nurse is reviewing data for the medical-surgical unit. The nurse notices an increase in postoperative infections from Aspergillus. Which type of health care–associated infection will the nurse report? a. Vector b. Exogenous c. Endogenous d. Suprainfection 18. The patient has contracted a urinary tract infection (UTI) while in the hospital. Which action will most likely increase the risk of a patient contracting a UTI? a. Reusing the patient’s graduated receptacle to empty the drainage bag. b. Allowing the drainage bag port to touch the graduated receptacle. c. Emptying the urinary drainage bag at least once a shift. d. Irrigating the catheter infrequently. 19. Which nursing action will most likely increase a patient’s risk for developing a health care–associated infection? a. Uses surgical aseptic technique to suction an airway b. Uses a clean technique for inserting a urinary catheter c. Uses a cleaning stroke from the urinary meatus toward the rectum d. Uses a sterile bottled solution more than once within a 24-hour period 20. The nurse is caring for a patient in labor and delivery. When near completing an assessment of the patient’s cervix, the electronic infusion device being used on the intravenous (IV) infusion alarms. Which sequence of actions is most appropriate for the nurse to take? a. Complete the assessment, remove gloves, and silence the alarm. b. Discontinue the assessment, silence the alarm, and assess the intravenous site. c. Complete the assessment, remove gloves, wash hands, and assess the intravenous infusion. d. Discontinue the assessment, remove gloves, use hand gel, and assess the intravenous infusion. 21. The nurse is dressed and is preparing to care for a patient in the perioperative area. The nurse has scrubbed hands and has donned a sterile gown and gloves. Which action will indicate a break in sterile technique? a. Touching clean protective eyewear b. Standing with hands above waist area c. Accepting sterile supplies from the surgeon d. Staying with the sterile table once it is open 22. The nurse is caring for a patient with an incision. Which actions will best indicate an understanding of medical and surgical asepsis for a sterile dressing change? a. Donning clean goggles, gown, and gloves to dress the wound b. Donning sterile gown and gloves to remove the wound dressing c. Utilizing clean gloves to remove the dressing and sterile supplies for the new dressing d. Utilizing clean gloves to remove the dressing and clean supplies for the new dressing 23. The nurse is caring for a patient in the endoscopy area. The nurse observes the technician performing these tasks. Which observation will require the nurse to intervene? a. Washing hands after removing gloves b. Disinfecting endoscopes in the workroom c. Removing gloves to transfer the endoscope d. Placing the endoscope in a container for transfer 24. The nurse is caring for a patient who is at risk for infection. Which action by the nurse indicates correct understanding about standard precautions? a. Teaches the patient about good nutrition b. Dons gloves when wearing artificial nails c. Disposes an uncapped needle in the designated container d. Wears eyewear when emptying the urinary drainage bag 25. The nurse is caring for a patient who has just delivered a neonate. The nurse is checking the patient for excessive vaginal drainage. Which precaution will the nurse use? a. Contact b. Droplet c. Standard d. Protective environment 26. The nurse is caring for a patient in the hospital. The nurse observes the nursing assistive personnel (NAP) turning off the handle faucet with bare hands. Which professional practice principle supports the need for follow-up with the NAP? a. The nurse is responsible for providing a safe environment for the patient. b. Different scopes of practice allow modification of procedures. c. Allowing the water to run is a waste of resources and money. d. This is a key step in the procedure for washing hands. 27. The nurse is caring for a patient who becomes nauseated and vomits without warning. The nurse has contaminated hands. Which action is best for the nurse to take next? a. Wash hands with an antimicrobial soap and water. b. Clean hands with wipes from the bedside table. c. Use an alcohol-based waterless hand gel. d. Wipe hands with a dry paper towel. 28. The nurse is performing hand hygiene before assisting a health care provider with insertion of a chest tube. While washing hands, the nurse touches the sink. Which action will the nurse take next? a. Inform the health care provider and recruit another nurse to assist. b. Rinse and dry hands, and begin assisting the health care provider. c. Extend the handwashing procedure to 5 minutes. d. Repeat handwashing using antiseptic soap. 29. The nurse on the surgical team and the surgeon have completed a surgery. After donning gloves, gathering instruments, and placing in the transport carrier, what is the next step in handling the instruments used during the procedure? a. Sending to central sterile for cleaning and sterilization b. Sending to central sterile for cleaning and disinfection c. Sending to central sterile for cleaning and boiling d. Sending to central sterile for cleaning 30. The nurse is observing a family member changing a dressing for a patient in the home health environment. Which observation indicates the family member has a correct understanding of how to manage contaminated dressings? a. The family member places the used dressings in a plastic bag. b. The family member saves part of the dressing because it is clean. c. The family member removes gloves and gathers items for disposal. d. The family member wraps the used dressing in toilet tissue before placing in trash. 31. The nurse is caring for a group of patients. Which patient will the nurse see first? a. A patient with Clostridium difficile in droplet precautions b. A patient with tuberculosis in airborne precautions c. A patient with MRSA infection in contact precautions d. A patient with a lung transplant in protective environment precautions 32. The home health nurse is teaching a patient and family about hand hygiene in the home. Which situation will cause the nurse to emphasize washing hands before and after? a. Shaking hands b. Performing treatments c. Opening the refrigerator d. Working on a computer 33. The surgical mask the perioperative nurse is wearing becomes moist. Which action will the perioperative nurse take next? a. Apply a new mask. b. Reapply the mask after it air-dries. c. Change the mask when relieved by next shift. d. Do not change the mask if the nurse is comfortable. 34. The nurse is caring for a patient on contact precautions. Which action will be mostappropriate to prevent the spread of disease? a. Place the patient in a room with negative airflow. b. Wear a gown, gloves, face mask, and goggles for interactions with the patient. c. Transport the patient safely and quickly when going to the radiology department. d. Use a dedicated blood pressure cuff that stays in the room and is used for that patient only. 35. The nurse is caring for a patient who has cultured positive for Clostridium difficile. Which action will the nurse take next? a. Instruct assistive personnel to use soap and water rather than sanitizer. b. Wear an N95 respirator when entering the patient room. c. Place the patient on droplet precautions. d. Teach the patient cough etiquette. 36. The nurse is changing linens for a postoperative patient and feels a prick in the left hand. A nonactivated safe needle is noted in the linens. For which condition is the nurse most at risk? a. Diphtheria b. Hepatitis B c. Clostridium difficile d. Methicillin-resistant Staphylococcus aureus 37. The nurse is caring for a patient who has a bloodborne pathogen. The nurse splashes blood above the glove to intact skin while discontinuing an intravenous (IV) infusion. Which step(s) will the nurse take next? a. Obtain an alcohol swab, remove the blood with an alcohol swab, and continue care. b. Immediately wash the site with soap and running water, and seek guidance from the manager. c. Do nothing; accidentally getting splashed with blood happens frequently and is part of the job. d. Delay washing of the site until the nurse is finished providing care to the patient. 38. Which process will be required after exposure of a nurse to blood by a cut from a used scalpel in the operative area? a. Placing the scalpel in a needle safe container b. Testing the patient and offering treatment to the nurse c. Removing sterile gloves and disposing of in kick bucket d. Providing a medical evaluation of the nurse to the manager 39. The nurse is caring for a patient who needs a protective environment. The nurse has provided the care needed and is now leaving the room. In which order will the nurse remove the personal protective equipment, beginning with the first step? 1. Remove eyewear/face shield and goggles. 2. Perform hand hygiene, leave room, and close door. 3. Remove gloves. 4. Untie gown, allow gown to fall from shoulders, and do not touch outside of gown; dispose of properly. 5. Remove mask by strings; do not touch outside of mask. 6. Dispose of all contaminated supplies and equipment in designated receptacles. a. 3, 1, 4, 5, 6, 2 b. 1, 4, 5, 3, 6, 2 c. 1, 4, 5, 3, 2, 6 d. 3, 1, 4, 5, 2, 6 40. The nurse manager is evaluating current infection control data for the intensive care unit. The nurse compares past patient data with current data to look for trends. The nurse manager examines the infection chain for possible solutions. In which order will the nurse arrange the items for the infection chain beginning with the first step? 1. A mode of transmission 2. An infectious agent or pathogen 3. A susceptible host 4. A reservoir or source for pathogen growth 5. A portal of entry to a host 6. A portal of exit from the reservoir a. 3, 2, 4, 1, 5, 6 b. 1, 3, 5, 4, 6, 2 c. 4, 2, 1, 6, 3, 5 d. 2, 4, 6, 1, 5, 3 MULTIPLE RESPONSE 1. The nurse is caring for a patient in protective environment. Which actions will the nurse take? (Select all that apply.) a. Wear an N95 respirator when entering the patient’s room. b. Maintain airflow rate greater than 12 air exchanges/hr. c. Place in special room with negative-pressure airflow. d. Open drapes during the daytime. e. Listen to the patient’s interests. f. Place dried flowers in a plastic vase. , D, E 2. The nurse is assessing a new patient admitted to home health. Which questions will be mostappropriate for the nurse to ask to determine the risk of infection? (Select all that apply.) a. “Can you explain the risk for infection in your home?” b. “Have you traveled outside of the United States?” c. “Will you demonstrate how to wash your hands?” d. “What are the signs and symptoms of infection?” e. “Are you able to walk to the mailbox?” f. “Who runs errands for you?” , B, C, D 3. The circulating nurse in the operating room is observing the surgical technologist while applying a sterile gown and gloves to care for a patient having an appendectomy. Which behaviors indicate to the nurse that the procedure by the surgical technologist is correct? (Select all that apply.) a. Ties the back of own gown b. Touches only the inside of gown c. Slips arms into arm holes simultaneously d. Extended fingers fully into both of the gloves e. Uses hands covered by sleeves to open gloves f. Applies surgical cap and face mask in the operating suite , C, D, E 4. The nurse is preparing to insert a urinary catheter. The nurse is using open gloving to apply the sterile gloves. Which steps will the nurse take? (Select all that apply.) a. While putting on the first glove, touch only the outside surface of the glove. b. With gloved dominant hand, slip fingers underneath second glove cuff. c. Remove outer glove package by tearing the package open. d. Lay glove package on clean flat surface above waistline. e. Glove the dominant hand of the nurse first. f. After second glove is on, interlock hands. , D, E, F 5. The nurse has received a report from the emergency department that a patient with tuberculosis will be coming to the unit. Which items will the nurse need to care for this patient? (Select all that apply.) a. Private room b. Negative-pressure airflow in room c. Surgical mask, gown, gloves, eyewear d. N95 respirator, gown, gloves, eyewear e. Communication signs for droplet precautions f. Communication signs for airborne precautions , B, D, F 6. The nurse and the student nurse are caring for two different patients on the medical-surgical unit. One patient is in airborne precautions, and one is in contact precautions. The nurse explains to the student different interventions for care. Which information will the nurse include in the teaching session? (Select all that apply.) a. Dispose of supplies to prevent the spread of microorganisms. b. Wash hands before entering and leaving both of the patients’ rooms. c. Be consistent in nursing interventions since there is only one difference in the precautions. d. Apply the knowledge the nurse has of the disease process to prevent the spread of microorganisms. e. Have patients in airborne precautions wear a mask during transportation to other departments. f. Check the working order of the negative-pressure room for the airborne precaution patient on admission and at discharge. , B, D, E Chapter 30: Vital Signs 1. A patient has a head injury and damages the hypothalamus. Which vital sign will the nurse monitor most closely? a. Pulse b. Respirations c. Temperature d. Blood pressure 2. A patient presents with heatstroke. The nurse uses cool packs, cooling blanket, and a fan. Which technique is the nurse using when the fan produces heat loss? a. Radiation b. Conduction c. Convection d. Evaporation 3. The patient has a temperature of 105.2° F. The nurse is attempting to lower temperature by providing tepid sponge baths and placing cool compresses in strategic body locations. Which technique is the nurse using to lower the patient’s temperature? a. Radiation b. Conduction c. Convection d. Evaporation 4. A nurse is focusing on temperature regulation of newborns and infants. Which action will the nurse take? a. Apply just a diaper. b. Double the clothing. c. Place a cap on their heads. d. Increase room temperature to 90 degrees. 5. The nurse is working the night shift on a surgical unit and is making 4:00 AM rounds. The nurse notices that the patient’s temperature is 96.8° F (36° C), whereas at 4:00 PM the preceding day, it was 98.6° F (37° C). What should the nurse do? a. Call the health care provider immediately to report a possible infection. b. Administer medication to lower the temperature further. c. Provide another blanket to conserve body temperature. d. Realize that this is a normal temperature variation. 6. The nurse is caring for a patient who has a temperature reading of 100.4° F (38° C). The patient’s last two temperature readings were 98.6° F (37° C) and 96.8° F (36° C). Which action will the nurse take? a. Wait 30 minutes and recheck the patient’s temperature. b. Assume that the patient has an infection and order blood cultures. c. Encourage the patient to move around to increase muscular activity. d. Be aware that temperatures this high are harmful and affect patient safety. 7. A patient is pyrexic. Which piece of equipment will the nurse obtain to monitor this condition? a. Stethoscope b. Thermometer c. Blood pressure cuff d. Sphygmomanometer 8. The nurse is caring for a patient who has an elevated temperature. Which principle will the nurse consider when planning care for this patient? a. Hyperthermia and fever are the same thing. b. Hyperthermia is an upward shift in the set point. c. Hyperthermia occurs when the body cannot reduce heat production. d. Hyperthermia results from a reduction in thermoregulatory mechanisms. 9. The patient with heart failure is restless with a temperature of 102.2° F (39° C). Which action will the nurse take? a. Place the patient on oxygen. b. Encourage the patient to cough. c. Restrict the patient’s fluid intake. d. Increase the patient’s metabolic rate. 10. The patient requires temperatures to be taken every 2 hours. Which task will the nurse assign to an RN? a. Using appropriate route and device b. Assessing changes in body temperature c. Being aware of the usual values for the patient d. Obtaining temperature measurement at ordered frequency 11. The patient requires routine temperature assessment but is confused, easily agitated, and has a history of seizures. Which route will the nurse use to obtain the patient’s temperature? a. Oral b. Rectal c. Axillary d. Tympanic 12. The patient is being admitted to the emergency department following a motor vehicle accident. The patient’s jaw is broken with several broken teeth. The patient is ashen, has cool skin, and is diaphoretic. Which route will the nurse use to obtain an accurate temperature reading? a. Oral b. Axillary c. Tympanic d. Temporal 13. The nurse is caring for an infant and is obtaining the patient’s vital signs. Which artery will the nurse use to best obtain the infant’s pulse? a. Radial b. Brachial c. Femoral d. Popliteal 14. The patient is found to be unresponsive and not breathing. Which pulse site will the nurse use? a. Radial b. Apical c. Carotid d. Brachial 15. The nurse needs to obtain a radial pulse from a patient. What must the nurse do to obtain a correct measurement? a. Place the tips of the first two fingers over the groove along the thumb side of the patient’s wrist. b. Place the tips of the first two fingers over the groove along the little finger side of the patient’s wrist. c. Place the thumb over the groove along the little finger side of the patient’s wrist. d. Place the thumb over the groove along the thumb side of the patient’s wrist. 16. The nurse is assessing the patient’s respirations. Which action by the nurse is mostappropriate? a. Inform the patient that she is counting respirations. b. Do not touch the patient until completed. c. Obtain without the patient knowing. d. Estimate respirations. 17. The patient’s blood pressure is 140/60. Which value will the nurse record for the pulse pressure? a. 60 b. 80 c. 140 d. 200 18. The nurse reviews the laboratory results for a patient and determines the viscosity of the blood is thick. Which laboratory result did the nurse check? a. Arterial blood gas b. Blood culture c. Hematocrit d. Potassium 19. The patient is being admitted to the emergency department with reports of shortness of breath. The patient has had chronic lung disease for many years but still smokes. What will the nurse do? a. Allow the patient to breathe into a paper bag. b. Use oxygen cautiously in this patient. c. Administer high levels of oxygen. d. Give CO2 via mask. 20. A nurse is reviewing capnography results for adult patients. Which value will cause the nurse to follow up? a. 35 mm Hg b. 40 mm Hg c. 45 mm Hg d. 50 mm Hg 21. The nurse is caring for a patient who has a pulse rate of 48. His blood pressure is within normal limits. Which finding will help the nurse determine the cause of the patient’s low heart rate? a. The patient has a fever. b. The patient has possible hemorrhage or bleeding. c. The patient has chronic obstructive pulmonary disease (COPD). d. The patient has calcium channel blockers or digitalis medication prescriptions. 22. The patient was found unresponsive in an apartment and is being brought to the emergency department. The patient has arm, hand, and leg edema, temperature is 95.6° F, and hands are cold secondary to a history of peripheral vascular disease. It is reported that the patient has a latex allergy. What should the nurse do to quickly measure the patient’s oxygen saturation? a. Attach a finger probe to the patient’s index finger. b. Place a nonadhesive sensor on the patient’s earlobe. c. Attach a disposable adhesive sensor to the bridge of the patient’s nose. d. Place the sensor on the same arm that the electronic blood pressure cuff is on. 23. The patient is admitted with shortness of breath and chest discomfort. Which laboratory value could account for the patient’s symptoms? a. Red blood cell count of 5.0 million/mm3 b. Hemoglobin level of 8.0 g/100 mL c. Hematocrit level of 45% d. Pulse oximetry of 95% 24. A nurse reviews blood pressures of several patients. Which finding will the nurse report as prehypertension? a. 98/50 in a 7-year-old child b. 115/70 in an infant c. 120/80 in a middle-aged adult d. 146/90 in an older adult 25. The nurse is providing a blood pressure clinic for the community. Which group will the nursemost likely address? a. Non-Hispanic Caucasians b. European Americans c. African-Americans d. Asian Americans 26. A nurse is caring for a patient who smokes and drinks caffeine. Which point is important for the nurse to understand before assessing the patient’s blood pressure (BP)? a. Smoking increases BP for up to 3 hours. b. Caffeine increases BP for up to 15 minutes. c. Smoking result in vasoconstriction, falsely elevating BP. d. Caffeine intake should not have occurred 30 to 40 minutes before BP measurement. 27. When taking the pulse of an infant, the nurse notices that the rate is 145 beats/min and the rhythm is regular. How should the nurse interpret this finding? a. This is normal for an infant. b. This is too fast for an infant. c. This is too slow for an infant. d. This is not a rate for an infant but for a toddler. 28. The nurse is caring for an older-adult patient and notes that the temperature is 96.8° F (36° C). How will the nurse interpret this finding? a. The patient has hyperthermia. b. The patient has a normal temperature. c. The patient is suffering from hypothermia. d. The patient is demonstrating increased metabolism. 29. When assessing the temperature of newborns and children, the nurse decides to utilize a temporal artery thermometer. What is the rationale for the nurse’s action? a. It is not affected by skin moisture. b. It has no risk of injury to patient or nurse. c. It reflects rapid changes in radiant temperature. d. It is accurate even when the forehead is covered with hair. 30. The nurse is caring for a small child and needs to obtain vital signs. Which site choice from the nursing assistive personnel (NAP) will cause the nurse to praise the NAP? a. Ulnar site b. Radial site c. Brachial site d. Femoral site 31. The nurse is caring for a newborn infant in the hospital nursery and notices that the infant is breathing rapidly but is pink, warm, and dry. Which normal respiratory rate will the nurse consider when planning care for this newborn? a. 30 to 60 b. 22 to 28 c. 16 to 20 d. 10 to 15 32. The nurse is preparing to obtain an oxygen saturation reading on a toddler. Which action will the nurse take? a. Secure the sensor to the toddler’s earlobe. b. Determine whether the toddler has a latex allergy. c. Place the sensor on the bridge of the toddler’s nose. d. Overlook variations between an oximeter pulse rate and the toddler’s pulse rate. 33. The nurse is preparing to assess the blood pressure of a 3-year-old. How should the nurse proceed? a. Use the diaphragm portion of the stethoscope to detect Korotkoff sounds. b. Obtain the reading before the child has a chance to “settle down.” c. Choose the cuff that says “Child” instead of “Infant.” d. Explain the procedure to the child. 34. A nurse is caring for a group of patients. Which patient will the nurse see first? a. A crying infant with P-165 and R-54 b. A sleeping toddler with P-88 and R-23 c. A calm adolescent with P-95 and R-26 d. An exercising adult with P-108 and R-24 35. The nurse is caring for a patient who is being discharged from the hospital after being treated for hypertension. The patient is instructed to take blood pressure 3 times a day and to keep a record of the readings. The nurse recommends that the patient purchase a portable electronic blood pressure device. Which other information will the nurse share with the patient? a. You can apply the cuff in any manner. b. You will need to recalibrate the machine. c. You can move your arm during the reading. d. You will need to use a stethoscope properly. 36. The nurse is caring for a patient who reports feeling light-headed and “woozy.” The nurse checks the patient’s pulse and finds that it is irregular. The patient’s blood pressure is 100/72. It was 113/80 an hour earlier. What should the nurse do? a. Apply more pressure to the radial artery to feel pulse. b. Perform an apical/radial pulse assessment. c. Call the health care provider immediately. d. Obtain arterial blood gases. 37. A nurse is caring for a group of patients. Which patient will the nurse see first? a. A 17-year-old male who has just returned from outside “for a smoke” who needs a temperature taken b. A 20-year-old male postoperative patient whose blood pressure went from 128/70 to 100/60 c. A 27-year-old male patient reporting pain whose blood pressure went from 124/70 to 130/74 d. An 87-year-old male suspected of hypothermia whose temperature is below normal 38. The health care provider prescription reads “Metoprolol (Lopressor) 50 mg PO daily. Do not give if blood pressure is less than 100 mm Hg systolic.” The patient’s blood pressure is 92/66. The nurse does not give the medication. Which action should the nurse take? a. Documents that the medication was not given because of low blood pressure b. Does not inform the health care provider that the medication was held c. Does not tell the patient what the blood pressure is d. Documents only what the blood pressure was. 39. After taking the patient’s temperature, the nurse documents the value and the route used to obtain the reading. What is the reason for the nurse’s action? a. Temperatures vary depending on the route used. b. Temperatures are readings of core measurements. c. Rectal temperatures are cooler than when taken orally. d. Axillary temperatures are higher than oral temperatures. 40. When taking an adult blood pressure, the onset of the sound the nurse hears is at 138, the muffled sound the nurse hears is at 70, and the disappearance of the sound the nurse hears is at 62. How should the nurse record this finding? a. 68 b. 76 c. 138/62 d. 138/70 41. The nursing assistive personnel (NAP) is taking vital signs and reports that a patient’s blood pressure is abnormally low. What should the nurse do next? a. Ask the NAP retake the blood pressure. b. Instruct the NAP to assess the patient’s other vital signs. c. Disregard the report and have it rechecked at the next scheduled time. d. Retake the blood pressure personally and assess the patient’s condition. MULTIPLE RESPONSE 1. A nurse is working in the intensive care unit and must obtain core temperatures on patients. Which sites can be used to obtain a core temperature? (Select all that apply.) a. Rectal b. Tympanic c. Esophagus d. Temporal artery e. Pulmonary artery , C, E 2. The patient has new-onset restlessness and confusion. Pulse rate is elevated, as is respiratory rate. Oxygen saturation is 94%. The nurse ignores the pulse oximeter reading and calls the health care provider for orders because the pulse oximetry reading is inaccurate. Which factors can cause inaccurate pulse oximetry readings? (Select all that apply.) a. O2 saturations (SaO2) > 70% b. Carbon monoxide inhalation c. Hypothermic fingers d. Intravascular dyes e. Nail polish f. Jaundice , C, D, E, F 3. The nurse is assessing the patient and family for probable familial causes of the patient’s hypertension. The nurse begins by analyzing the patient’s personal history, as well as family history and current lifestyle situation. Which findings will the nurse consider to be risk factors? (Select all that apply.) a. Obesity b. Cigarette smoking c. Recent weight loss d. Heavy alcohol intake e. Regular exercise sessions , B, D 4. The patient is being encouraged to purchase a portable automatic blood pressure device to monitor blood pressure at home. Which information will the nurse present as benefits for this type of treatment? (Select all that apply.) a. Patients can actively participate in their treatment. b. Self-monitoring helps with compliance and treatment. c. The risk of obtaining an inaccurate reading is decreased. d. Blood pressures can be obtained if pulse rates become irregular. e. Patients can provide information about patterns to health care providers. , B, E 5. A nurse is teaching the staff about alterations in breathing patterns. Which information will the nurse include in the teaching session? (Select all that apply.) a. Apnea—no respirations b. Tachypnea—regular, rapid respirations c. Kussmaul’s—abnormally deep, regular, fast respirations d. Hyperventilation—labored, increased in depth and rate respirations e. Cheyne-Stokes—abnormally slow and depressed ventilation respirations f. Biot’s—irregular with alternating periods of apnea and hyperventilation respirations , B, C MATCHING A nurse is assessing results of vital signs for a group of patients. Match the condition to the assessment findings the nurse is reviewing. a. Patient’s temperature is 113° F (45° C) with hot, dry skin. b. Patient’s blood pressure sitting is 130/60 and 110/40 standing. c. Patient’s pulse is 110 beats/min. d. Patient’s temperature is 93.2° F (34° C). e. Patient’s blood pressure went from 126/76 to 90/50. 1. Hypothermia 2. Shock/Hypotension 3. Heatstroke 4. Orthostatic hypotension 5. Tachycardia 1.ANS DIF:Understand (comprehension)REF:491 | 499 | 505 OBJ: Accurately assess body temperature, pulse, respirations, oxygen saturation, and blood pressure. TOP: Assessment MSC: Physiological Adaptation 2.ANS:EDIF:Understand (comprehension)REF:491 | 499 | 505 OBJ: Accurately assess body temperature, pulse, respirations, oxygen saturation, and blood pressure. TOP: Assessment MSC: Physiological Adaptation 3.ANS:ADIF:Understand (comprehension)REF:491 | 499 | 505 OBJ: Accurately assess body temperature, pulse, respirations, oxygen saturation, and blood pressure. TOP: Assessment MSC: Physiological Adaptation 4.ANS:BDIF:Understand (comprehension)REF:491 | 499 | 505 OBJ: Accurately assess body temperature, pulse, respirations, oxygen saturation, and blood pressure. TOP: Assessment MSC: Physiological Adaptation 5.ANS:CDIF:Understand (comprehension)REF:491 | 499 | 505 OBJ: Accurately assess body temperature, pulse, respirations, oxygen saturation, and blood pressure. TOP: Assessment MSC: Physiological Adaptation Chapter 40: Hygiene 1. A nurse is preparing to provide hygiene care. Which principle should the nurse consider when planning hygiene care? a. Hygiene care is always routine and expected. b. No two individuals perform hygiene in the same manner. c. It is important to standardize a patient’s hygienic practices. d. During hygiene care do not take the time to learn about patient needs. 2. A patient’s hygiene schedule of bathing and brushing teeth is largely influenced by family customs. For which age group is the nurse most likely providing care? a. Adolescent b. Preschooler c. Older adult d. Adult 3. The patient has been diagnosed with diabetes. When admitted, the patient is unkempt and is in need of a bath and foot care. When questioned about hygiene habits, the nurse learns the patient takes a bath once a week and a sponge bath every other day. To provide ultimate care for this patient, which principle should the nurse keep in mind? a. Patients who appear unkempt place little importance on hygiene practices. b. Personal preferences determine hygiene practices and are unchangeable. c. The patient’s illness may require teaching of new hygiene practices. d. All cultures value cleanliness with the same degree of importance. 4. The nurse is caring for a patient who refuses to bathe in the morning. When asked why, the patient says “I always bathe in the evening.” Which action by the nurse is best? a. Defer the bath until evening and pass on the information to the next shift. b. Tell the patient that daily morning baths are the “normal” routine. c. Explain the importance of maintaining morning hygiene practices. d. Cancel hygiene for the day and attempt again in the morning. 5. A nurse is completing an assessment of the patient. Which principle is a priority? a. Foot care will always be important. b. Daily bathing will always be important. c. Hygiene needs will always be important. d. Critical thinking will always be important. 6. When providing hygiene for an older-adult patient, the nurse closely assesses the skin. What is the rationale for the nurse’s action? a. Outer skin layer becomes more resilient. b. Less frequent bathing may be required. c. Skin becomes less subject to bruising. d. Sweat glands become more active. 7. The nurse is bathing a patient and notices movement in the patient’s hair. Which action will the nurse take? a. Use gloves to inspect the hair. b. Apply a lindane-based shampoo immediately. c. Shave the hair off of the patient’s head. d. Ignore the movement and continue. 8. The patient has been brought to the emergency department following a motor vehicle accident. The patient is unresponsive. The driver’s license states that glasses are needed to operate a motor vehicle, but no glasses were brought in with the patient. Which action should the nurse take next? a. Stand to the side of the patient’s eye and observe the cornea. b. Conclude that the glasses were lost during the accident. c. Notify the ambulance personnel for missing glasses. d. Ask the patient where the glasses are. 9. A nurse is assessing a patient’s skin. Which patient is most at risk for impaired skin integrity? a. A patient who is afebrile b. A patient who is diaphoretic c. A patient with strong pedal pulses d. A patient with adequate skin turgor 10. The nurse is caring for a patient who is immobile. The nurse frequently checks the patient for impaired skin integrity. What is the rationale for the nurse’s action? a. Inadequate blood flow leads to decreased tissue ischemia. b. Patients with limited caloric intake develop thicker skin. c. Pressure reduces circulation to affected tissue. d. Verbalization of skin care needs is decreased. 11. The nurse is caring for a patient who has diabetes mellitus and circulatory insufficiency, with peripheral neuropathy and urinary incontinence. On which areas does the nurse focus care? a. Decreased pain sensation and increased risk of skin impairment b. Decreased caloric intake and accelerated wound healing c. High risk for skin infection and low saliva pH level d. High risk for impaired venous return and dementia 12. The nurse is caring for a patient who has undergone surgery for a broken leg and has a cast in place. What should the nurse do to prevent skin impairment? a. Assess surfaces exposed to the edges of the cast for pressure areas. b. Keep the patient’s blood pressure low to prevent overperfusion of tissue. c. Do not allow turning in bed because that may lead to redislocation of the leg. d. Restrict the patient’s dietary intake to reduce the number of times on the bedpan. 13. Which action by the nurse will be the most important for preventing skin impairment in a mobile patient with local nerve damage? a. Insert an indwelling urinary catheter. b. Limit caloric and protein intake. c. Turn the patient every 2 hours. d. Assess for pain during a bath. 14. After performing foot care, the nurse checks the medical record and discovers that the patient has a foot disorder caused by a virus. Which condition did the nurse most likely observe? a. Corns b. A callus c. Plantar warts d. Athlete’s foot 15. The nurse is caring for a patient who is reporting severe foot pain due to corns. The patient has been using oval corn pads to self-treat the corns, but they seem to be getting worse. Which information will the nurse share with the patient? a. Corn pads are an adequate treatment and should be continued. b. The patient should avoid soaking the feet before using a pumice stone. c. Depending on severity, surgery may be needed to remove the corns. d. Tighter shoes would help to compress the corns and make them smaller. 16. The patient is diagnosed with athlete’s foot (tinea pedis). The patient says that he is relieved because it is only athlete’s foot, and it can be treated easily. Which information should the nurse consider when formulating a response to the patient? a. Contagious with frequent recurrences b. Helpful to air-dry feet after bathing c. Treated with salicylic acid d. Caused by lice 17. When assessing a patient’s feet, the nurse notices that the toenails are thick and separated from the nail bed. What does the nurse most likely suspect is the cause of this condition? a. Fungi b. Friction c. Nail polish d. Nail polish remover 18. The nurse is providing education about the importance of proper foot care to a patient who has diabetes mellitus. Which primary goal is the nurse trying to achieve? a. Prevention of plantar warts b. Prevention of foot fungus c. Prevention of neuropathy d. Prevention of amputation 19. The nurse is providing oral care to an unconscious patient and notes that the patient has extremely bad breath. Which term will the nurse use when reporting to the oncoming shift? a. Cheilitis b. Halitosis c. Glossitis d. Dental caries 20. The nurse is caring for a patient with diabetes. Which task will the nurse assign to the nursing assistive personnel? a. Providing nail care b. Teaching foot care c. Making an occupied bed d. Determining aspiration risk 21. The patient is being treated for cancer with weekly radiation therapy to the head and chemotherapy treatments. Which assessment is the priority? a. Feet b. Nail beds c. Perineum d. Oral cavity 22. The nurse is providing oral care to an unconscious patient. Which action should the nurse take? a. Moisten the mouth using lemon-glycerin sponges. b. Hold the patient’s mouth open with gloved fingers. c. Use foam swabs to help remove plaque. d. Suction the oral cavity. 23. The nurse is teaching the patient about flossing and oral hygiene. Which instruction will the nurse include in the teaching session? a. Using waxed floss prevents bleeding. b. Flossing removes plaque and tartar from the teeth. c. Performing flossing at least 3 times a day is beneficial. d. Applying toothpaste to the teeth before flossing is harmful. 24. The nurse is teaching the parents of a child who has head lice (pediculosis capitis). Which information will the nurse include in the teaching session? a. Treatment is use of regular shampoo. b. Products containing lindane are most effective. c. Head lice may spread to furniture and other people. d. Manual removal is not a realistic option as treatment. 25. A patient has scaling of the scalp. Which term will the nurse use to report this finding to the oncoming staff? a. Dandruff b. Alopecia c. Pediculosis d. Xerostomia 26. A nurse is providing a bath. In which order will the nurse clean the body, beginning with the first area? 1. Face 2. Eyes 3. Perineum 4. Arm and chest 5. Hands and nails 6. Back and buttocks 7. Abdomen and legs a. 1, 2, 5, 4, 7, 6, 3 b. 2, 1, 4, 5, 7, 3, 6 c. 2, 1, 5, 4, 6, 7, 3 d. 1, 2, 4, 5, 3, 7, 6 27. The nurse is caring for a patient who has multiple ticks on lower legs and body. What should the nurse do to rid the patient of ticks? a. Use blunt tweezers and pull upward with steady pressure. b. Burn the ticks with a match or small lighter. c. Allow the ticks to drop off by themselves. d. Apply miconazole and cover with plastic. 28. The nurse is providing oral care to a patient. In which order will the nurse clean the oral cavity, starting with the first area? 1. Roof of mouth, gums, and inside cheek 2. Chewing and inner tooth surfaces 3. Outer tooth surfaces 4. Tongue a. 4, 1, 3, 2 b. 3, 2, 4, 1 c. 2, 3, 1, 4 d. 1, 4, 2, 3 Oral care is provided in the following sequence: Clean chewing and inner tooth surfaces first. Clean outer tooth surfaces. Moisten brush with chlorhexidine rinse to rinse. Use toothette to clean roof of mouth, gums, and inside cheeks. Gently brush tongue but avoid stimulating gag reflex. Rinse. DIF:Understand (comprehension)REF:866 OBJ:Successfully perform hygiene procedures for the care of the skin, perineum, feet and nails, mouth, eyes, ears, and nose.TOP:Implementation MSC: Basic Care and Comfort 29. The nurse is caring for an older-adult patient with Alzheimer’s disease who is ambulatory but requires total assistance with activities of daily living (ADLs). The nurse notices that the patient is edentulous. Which area should the nurse assess? a. Assess oral cavity. b. Assess room for drafts. c. Assess ankles for edema. d. Assess for reduced sensations. 30. A self-sufficient bedridden patient is unable to reach all body parts. Which type of bath will the nurse assign to the nursing assistive personnel? a. Bag bath b. Sponge bath c. Partial bed bath d. Complete bed bath 31. The nurse is preparing to provide a complete bed bath to an unconscious patient. The nurse decides to use a bag bath. In which order will the nurse clean the body, starting with the first area? 1. Neck, shoulders, and chest 2. Abdomen and groin/perineum 3. Legs, feet, and web spaces 4. Back of neck, back, and then buttocks 5. Both arms, both hands, web spaces, and axilla a. 5, 1, 2, 3, 4 b. 1, 5, 2, 3, 4 c. 1, 5, 2, 4, 3 d. 5, 1, 2, 4, 3 Use all six chlorhexidene gluconate (CHG) cloths in the following order: 1. Cloth 1: Neck, shoulders, and chest 2. Cloth 2: Both arms, both hands, web spaces, and axilla 3. Cloth 3: Abdomen and then groin/perineum 4. Cloth 4: Right leg, right foot, and web spaces 5. Cloth 5: Left leg, left foot, and web spaces 6. Cloth 6: Back of neck, back, and then buttocks DIF:Understand (comprehension)REF:860 OBJ: Discuss different approaches used in maintaining a patient’s comfort and safety during hygiene care. TOP: Implementation MSC: Basic Care and Comfort 32. The female nurse is caring for a male patient who is uncircumcised but not ambulatory and has full function of all extremities. The nurse is providing the patient with a partial bed bath. How should perineal care be performed for this patient? a. Should be postponed because it may cause embarrassment b. Should be unnecessary because the patient is uncircumcised c. Should be done by the patient d. Should be done by the nurse 33. A nursing assistive personnel (NAP) is providing AM care to patients. Which action by the NAP will require the nurse to intervene? a. Not offering a backrub to a patient with fractured ribs b. Not offering to wash the hair of a patient with neck trauma c. Turning off the television while giving a backrub to the patient d. Turning patient’s head with neck injury to side when giving oral care 34. A nurse is providing AM care to patients. Which action will the nurse take? a. Soaks feet of patient with peripheral vascular disease b. Applies CHG solution to wash perineum of patient with a stroke c. Cleanses eye from outer canthus to inner canthus of patient with diabetes d. Uses long, firm stroke to wash legs of patient with blood-clotting disorder 35. The nurse is providing a complete bed bath to a patient using a commercial bath cleansing pack (bag bath). What should the nurse do? a. Rinse thoroughly. b. Allow the skin to air-dry. c. Do not use a bath towel. d. Dry the skin with a towel. 36. A nurse is providing perineal care to a female patient. Which washing technique will the nurse use? a. Back to front b. In a circular motion c. From pubic area to rectum d. Upward from rectum to pubic area 37. The nurse is providing perineal care to an uncircumcised male patient. Which action will the nurse take? a. Leave the foreskin alone because there is little chance of infection. b. Retract the foreskin for cleansing and allow it to return on its own. c. Retract the foreskin and return it to its natural position when done. d. Leave the foreskin retracted. 38. Which instruction will the nurse provide to the nursing assistive personnel when providing foot care for a patient with diabetes? a. Do not place slippers on the patient’s feet. b. Trim the patient’s toenails daily. c. Report sores on the patient’s toes. d. Check the brachial artery. 39. The debilitated patient is resisting attempts by the nurse to provide oral hygiene. Which action will the nurse take next? a. Insert an oral airway. b. Place the patient in a flat, supine position. c. Use undiluted hydrogen peroxide as a cleaner. d. Quickly proceed while not talking to the patient. 40. A nurse is providing oral care to a patient with stomatitis. Which technique will the nurse use? a. Avoid commercial mouthwashes. b. Avoid normal saline rinses. c. Brush with a hard toothbrush. d. Brush with an alcohol-based toothpaste. 41. The nurse is teaching a patient about contact lens care. Which instructions will the nurse include in the teaching session? a. Use tap water to clean soft lenses. b. Wash and rinse lens storage case daily. c. Reuse storage solution for up to a week. d. Keep the lenses is a cool dry place when not being used. 42. The patient reports to the nurse about a perceived decrease in hearing. When the nurse examines the patient’s ear, a large amount of cerumen buildup at the entrance to the ear canal is observed. Which action will the nurse take next? a. Teach the patient how to use cotton-tipped applicators. b. Tell the patient to use a bobby pin to extract earwax. c. Apply gentle, downward retraction of the ear canal. d. Instill hot water into the ear canal to melt the wax. 43. The patient is being fitted with a hearing aid. In teaching the patient how to care for the hearing aid, which instructions will the nurse provide? a. Change the battery every day or as needed. b. Adjust the volume for a talking distance of 1 yard. c. Wear the hearing aid 24 hours per day except when sleeping. d. Avoid the use of hairspray, but aerosol perfumes are allowed. 44. The patient is reporting an inability to clear nasal passages. Which action will the nurse take? a. Use gentle suction to prevent tissue damage. b. Instruct patient to blow nose forcefully to clear the passage. c. Place a dry washcloth under the nose to absorb secretions. d. Insert a cotton-tipped applicator to the back of the nose. 45. A patient uses an in-the-canal hearing aid. Which assessment is a priority? a. Eyeglass usage b. Cerumen buildup c. Type of physical exercise d. Excessive moisture problems MULTIPLE RESPONSE 1. The nurse is caring for a patient with cognitive impairments. Which actions will the nurse take during AM care? (Select all that apply.) a. Administer ordered analgesic 1 hour before bath time. b. Increase the frequency of skin assessment. c. Reduce triggers in the environment. d. Keep the room temperature cool. e. Be as quick as possible. , C 2. The nurse is caring for a patient who has peripheral neuropathy. Which clinical manifestations does the nurse expect to find upon assessment? (Select all that apply.) a. Abnormal gait b. Foot deformities c. Absent or decreased pedal pulses d. Muscle wasting of lower extremities e. Decreased hair growth on legs and feet , B, D 3. A nurse is providing hygiene care to a bariatric patient using chlorhexidine gluconate (CHG) wipes. Which actions will the nurse take? (Select all that apply.) a. Do not rinse. b. Clean under breasts. c. Inform that the skin will feel sticky. d. Dry thoroughly between skin folds. e. Use two wipes for each area of the body. , B, C 4. Which patients will the nurse determine are in need of perineal care? (Select all that apply.) a. A patient with rectal and genital surgical dressings b. A patient with urinary and fecal incontinence c. A circumcised male who is ambulatory d. A patient who has an indwelling catheter e. A bariatric patient , B, D, E 5. The patient must stay in bed for a bed change. Which actions will the nurse implement? (Select all that apply.) a. Apply sterile gloves. b. Keep soiled linen close to uniform. c. Advise patient will feel a lump when rolling over. d. Turn clean pillowcase inside out over the hand holding it. e. Make a modified mitered corner with sheet, blanket, and spread. , D, E [Show More]

Last updated: 1 year ago

Preview 1 out of 188 pages

Add to cart

Instant download

document-preview

Buy this document to get the full access instantly

Instant Download Access after purchase

Add to cart

Instant download

Reviews( 0 )

$20.00

Add to cart

Instant download

Can't find what you want? Try our AI powered Search

OR

REQUEST DOCUMENT
77
0

Document information


Connected school, study & course


About the document


Uploaded On

Oct 10, 2020

Number of pages

188

Written in

Seller


seller-icon
Expert1

Member since 4 years

884 Documents Sold


Additional information

This document has been written for:

Uploaded

Oct 10, 2020

Downloads

 0

Views

 77

Document Keyword Tags

Recommended For You

Get more on TEST BANK »

$20.00
What is Browsegrades

In Browsegrades, a student can earn by offering help to other student. Students can help other students with materials by upploading their notes and earn money.

We are here to help

We're available through e-mail, Twitter, Facebook, and live chat.
 FAQ
 Questions? Leave a message!

Follow us on
 Twitter

Copyright © Browsegrades · High quality services·